Psychosocial Nursing Exam 1 Practice Questions

अब Quizwiz के साथ अपने होमवर्क और परीक्षाओं को एस करें!

Examples of this term include medications not used for the patient's diagnosis and therapeutic holds

Restraint

A nurse is caring for a client who has severe anxiety disorder and is in a state of panic in the dayroom. Which of the following actions should the nurse take? A) Speak to the client in a calm voice B) Leave the client to regain C) Encourage the client to express their feelings D) Place the client in restraints

A) Speak to the client in a calm voice

Which situation demonstrates the use of primary intervention related to crisis? A) teaching stress-reduction techniques to a first-year college student B) Assessing coping strategies used by a client who attempted suicide C) Implementation of suicide precautions for a depressed client D) Referring a client diagnosed with schizophrenia to a partial hospitalization program

A) Teaching stress-reduction techniques to a first-year college student

An experienced nurse says to a new graduate, "When you've practiced as long as I have, you automatically know how to take care of patients experiencing psychosis." Which factors should the new graduate consider when analyzing this comment? (Select all that apply.) a. The experienced nurse may have lost sight of patients' individuality, which may compromise the integrity of practice. b. New research findings should be integrated continuously into a nurse's practice to provide the most effective care. c. Experience provides mental health nurses with the essential tools and skills needed for effective professional practice. d. Experienced psychiatric nurses have learned the best ways to care for mentally ill patients through trial and error. e. An intuitive sense of patients' needs guides effective psychiatric nurses.

A) The experienced nurse may have lost sight of patients' individuality, which may compromise the integrity of practice. B) New research findings should be integrated continuously into a nurse's practice to provide the most effective care. Evidence-based practice involves using research findings and standards of care to provide the most effective nursing care. Evidence is continuously emerging, so nurses cannot rely solely on experience. The effective nurse also maintains respect for each patient as an individual. Overgeneralization compromises that perspective. Intuition and trial and error are unsystematic approaches to care.

A nurse driving home after work comes upon a serious automobile accident. The drive gets out of the car with no apparent physical injuries. Which assessment findings would the nurse expect from the driver immediately after this event? Select all that apply A) Rapid speech B) Fecal incontinence C) Long-term memory losses D) Trembling E) Difficulty using a cell phone

A) rapid speech D) Trembling E) Difficulty using a cell phone

This type of admission can only be initiated by a licensed provider, such as a licensed professional counselor

An Emergency Order of Detention

INTENTIONAL tort: A verbal threat to another individual that is perceived as harmful

Assault

Complete this analogy. NANDA: clinical judgment: NIC: _________________ a) patient outcomes. b) nursing home actions. c) diagnosis. d) symptoms.

B) Nursing home actions Analogies show parallel relationships. NANDA, the North American Nursing Diagnosis Association, identifies diagnostic statements regarding human responses to actual or potential health problems. These statements represent clinical judgments. NIC (Nursing Interventions Classification) identifies actions provided by nurses that enhance patient outcomes. Nursing care activities may be direct or indirect.

An ER nurse is assessing a client who has an anxiety disorder. The client is flushed, perspiring profusely, and is experiencing palpitations. The client begins to scream, "I am going to die! This is it! I am having a heart attack!" The nurse should determine the client's level of anxiety to be which of the following? A) Moderate B) Panic C) Severe D) Mild

B) Panic

A staff nurse completes orientation to a psychiatric unit. This nurse may expect an advanced practice nurse to perform which additional intervention? a) Conduct mental health assessments. b) Prescribe psychotropic medication. C)Establish therapeutic relationships. D) Individualize nursing care plans.

B) Prescribe psychotropic medication In most states, prescriptive privileges are granted to master's-prepared nurse practitioners and clinical nurse specialists who have taken special courses on prescribing medication. The nurse prepared at the basic level is permitted to perform mental health assessments, establish relationships, and provide individualized care planning.

A patient participated in psychotherapy weekly for 5 months. The therapist used free association, dream analysis, and facilitated transference to help the patient understand conflicts and foster change. Select the term that applies to this method. a. Rational-emotive behavior therapy b. Psychodynamic psychotherapy c. Cognitive-behavioral therapy d. Operant conditioning

B) Psychodynamic psychotherapy The techniques are aspects of psychodynamic psychotherapy. The distracters use other techniques.

A nurse is planning care for a client who has thoughts of suicide. Which of the following goals should the nurse include in the client's plan of care A) The client will identify positive aspects of others B) The client agrees to notify a staff member of thoughts of self-harm C) The client will engage in an independent diversional activity D) The client will not verbalize thoughts or feelings related to suicide

B) The client agrees to notify a staff member of thoughts of self-harm Safety = priority; what will keep our client the most safe?

INTENTIONAL tort: a physical offensive action towards another individual

Battery

This term refers to a nurse's duty to do what is best for a patient

Beneficence

Unintentional tort: the nurse's performance fails to meet the standard of care

Breach of duty

This right allows a patient the ability to spend time in an open space away from the rest of the unit

timeout

A nurse receives a call on a crisis intervention hotline from a client. Which of the following statement should the nurse identify as an overt statement indicating the client's risk for suicide? A) "Everything will be better soon" B) "Soon no one will have to worry about me" C) "There's no point in living any longer" D) "I want to donate my organs to help others"

C) "Theres no point in living any longer"

A nurse encounters an unfamiliar psychiatric disorder on a new patient's admission form. Which resource should the nurse consult to determine criteria used to establish this diagnosis? a) International Statistical Classification of Diseases and Related Health Problems (ICD-10) b) The ANA's Psychiatric-Mental Health Nursing Scope and Standards of Practice c) Diagnostic and Statistical Manual of Mental Disorders (DSM-V) d) A behavioral health reference manual

C) DSM-V The DSM-V gives the criteria used to diagnose each mental disorder. It is the official guideline for diagnosing psychiatric disorders. The distracters may not contain diagnostic criteria for a psychiatric illness.

A client undergoing diagnostic tests says, "Nothing is wrong with me except a stubborn chest cold." The spouse reports the client smokes, coughs daily, lost 15 pounds, and is easily fatigued. which defense mechanism is the client using? A) Displacement B) Regression C) Denial D) Projection

C) Denial

A nurse consistently encourages patient to do his or her own activities of daily living. If the patient is unable to complete an activity, the nurse helps until the patient is once again independent. This nurse's practice is most influenced by which theorist? a. Betty Neuman b. Patricia Benner c. Dorothea Orem d. Joyce Travelbee

C) Dorothea Orem Orem emphasizes the role of the nurse in promoting self-care activities of the patient; this has relevance to the seriously and persistently mentally ill patient.

Operant conditioning is part of the treatment plan to encourage speech in a child who is nearly mute. Which technique applies? a. Encourage the child to observe others talking. b. Include the child in small group activities. c. Give the child a small treat for speaking. d. Teach the child relaxation techniques.

C) Give the child a small treat for speaking Operant conditioning involves giving positive reinforcement for a desired behavior. Treats are rewards and reinforce speech through positive reinforcement.

A nurse is caring for a client who attempted suicide and refuses to sign a no-suicide contract. Which of the following actions should the nurse take when implementing suicide precautions? A) Assign the client to a private room B) Request that the dietary department provide the client with finger foods C) place the client on one-to-one observation D) Keep the door to the client's room closed

C) place the client on one-to-one observation

This type of admission is determined by a mental health judge

Court commit

A nurse is admitting a client following care in the emergency department for an intentional overdose of opioids. The client states, "I feel so alone. No one can help me." Which of the following responses by the nurse is therapeutic? A) "Lets finish your admission and then talk about your feelings" B) "How come you feel that no one can help you when you are receiving help now?" C) "Why do you feel that no one can help you?" D) "I would like to sit and talk with you"

D)"I would like to sit and talk with you" Never ask why questions

An adult says, "Most of the time I'm happy and feel good about myself. I have learned that what I get out of something is proportional to the effort I put into it." Which number on this mental health continuum should the nurse select?Mental Illness: 1, 2Mental Health: 3, 4, 5 a) 1 b) 2 c) 3 d) 4 e) 5

E) 5 The adult is generally happy and has an adequate self-concept. The statement indicates the adult is reality-oriented, works effectively, and has control over own behavior. Mental health does not mean that a person is always happy.

Examples of this term include keeping our promises, doing what is expected of us, performing our duties, and being trustworthy

Fidelity

This term refers to a patient's right to receive care that is appropriate to their mental health needs

Least restrictive environment

What are the co-morbidities/predispositions that can increase a patient's risk for suicide?

Male, chronic health condition, recent divorce, being above 50 years old

UNINTENTIONAL tort: unprofessional treatment of a patient that results in injury, suffering, or death with five elements to prove

Malpractice

UNINTENTIONAL tort: examples include failure to administer a medication, failure to provide discharge education, and failure to follow a health care provider's order

Negligence

This term represents the principle of truth telling

Veracity

This type of admission allows an individual the right to request and obtain a release

Voluntary admission

This term refers to preventing disclosure of information without a patient's consent

confidentiality

UNINTENTIONAL tort: examples include medical expenses, loss of wages, and permanent disability

damages

INTENTIONAL tort: examples of this tort include slander and libel

defamation of character

This term refers to a commitment to warn third parties when they may be harmed by a patient

duty to warn

Which INTENTIONAL tort includes not allowing a voluntarily admitted patient the right to leave

false imprisonment

This type of admission is the least restrictive

informal admission

INTENTIONAL tort: taking a photo of a patient without their permission

invasion of privacy

Examples of this term include fair distribution of resources, competing needs, rights and obligations, and potential conflicts with established legislation

justice

UNINTENTIONAL tort: a legal term for a cause-and-effect relationship

proximate cause

According to the Western scientific view of health, illness is the result of A. pathogens. B. energy blockage. C. spirit invasion. D. soul loss.

A. pathogens. Disease has a cause (e.g., pathogens, toxins) that creates the effect; disease can be observed and measured.

A potential problem for a client diagnosed with severe obsessive-compulsive disorder is: A. sleep disturbance. B. excessive socialization. C. command hallucinations. D. altered state of consciousness.

A. sleep disturbance. Clients who must engage in compulsive rituals for anxiety relief are rarely afforded relief for any prolonged period. The high anxiety level and need to perform the ritual may interfere with sleep.

A cultural characteristic that may be observed in a teenage, female Hispanic client in times of stress is to: A. suddenly tremble severely. B. exhibit stoic behavior. C. report both nausea and vomiting. D. laugh inappropriately.

A. suddenly tremble severely. Ataque de nervios (attack of the nerves) is a culture-bound syndrome that is seen in undereducated, disadvantaged females of Hispanic ethnicity.

A client is experiencing a panic attack. The nurse can be most therapeutic by: A. telling the client to take slow, deep breaths. B. verbalizing mild disapproval of the anxious behavior. C. asking the client what he means when he says "I am dying." D. offering an explanation about why the symptoms are occurring

A. telling the client to take slow, deep breaths. Slow diaphragmatic breathing can induce relaxation and reduce symptoms of anxiety. Often the nurse has to tell the client to "breathe with me" and keep the client focused on the task. The slower breathing also reduces the threat of hypercapnia with its attendant symptoms.

Carter experiences each of the following during his crisis. Which of the following describes phase IV of Caplan's phases of crisis? A. Carter experiences increased anxiety and feelings of extreme discomfort the day after the tornado. B. Carter comes to the crisis clinic complaining of depression and expresses that he does not want to go on living. C. Carter experiences a panic attack at his mother's home in a nearby town where he is staying after the tornado. D. Carter experiences anxiety symptoms the day after he was fired.

B) Carter comes to the crisis clinic complaining of depression and expresses that he does not want to go on living. This describes phase IV, which, if coping is ineffective, may lead to depression, confusion, violence, or suicidality. The other options describe phase II, phase III, and phase I in Caplan's phases of crisis.

It can be said that the onset of most anxiety disorders occurs: A. before the age of 20 years. B. before the age of 40 years. C. after the age of 40 years. D. scattered throughout the life span.

B. before the age of 40 years. Epidemiology reports indicate that the onset of most anxiety disorders occurs before age 40 years.

A client is diagnosed with generalized anxiety disorder (GAD). The nursing assessment supports this diagnosis when the client reports: A. that his symptoms started right after he was robbed at gunpoint. B. being so worried he hasn't been able to work for the last 12 months. C. that eating in public makes him extremely uncomfortable. D. repeatedly verbalizing his prayers helps him feel relaxed.

B. being so worried he hasn't been able to work for the last 12 months. GAD is characterized by symptomatology that lasts 6 months or longer.

A man continues to speak of his wife as though she were still alive, 3 years after her death. This behavior suggests the use of: A. altruism. B. denial. C. undoing. D. suppression.

B. denial. Denial involves escaping unpleasant reality by ignoring its existence.

The Eastern world view can be identified by the belief that A. one's identity is found in individuality. B. holds responsibility to family as central. C. time waits for no one .D. disease is a lack of harmony with the environment.

B. holds responsibility to family as central. The Eastern traditional world view is sociocentric. Individuals experience their selfhood and their lives as part of an interdependent web of relationships and expectations.

A client reporting gastric pain, tells the nurse, "I think my symptoms started when a neighbor cast a spell on me." The assessment the nurse can make is that the client A. has a major mental illness. B. is expressing a culture-bound illness. C. requires hospitalization to protect the neighbor. D. will probably not respond to Western medical treatment.

B. is expressing a culture-bound illness. Many culture-bound illnesses, such as ghost illness, or hwa byung, seem exotic or irrational to American nurses. Many of these illnesses cannot be understood within a Western medicine framework. Their causes, manifestations, and treatments do not make sense to nurses whose understanding is limited to a Western perspective on disease and illness.

Selective inattention is first noted when experiencing anxiety that is: A. mild. B. moderate. C. severe. D. panic.

B. moderate. When moderate anxiety is present, the individual's perceptual field is reduced and the client is not able to see the entire picture of events.

A teenager changes study habits to earn better grades after initially failing a test. This behavioral change is likely a result of: A. a rude awakening. B. normal anxiety. C. trait anxiety. D. altruism.

B. normal anxiety. Normal anxiety is a healthy life force needed to carry out the tasks of living and striving toward goals. It prompts constructive actions.

A 20-year-old was sexually molested at age 10, but he can no longer remember the incident. The ego defense mechanism in use is: A. projection. B. repression. C. displacement. D. reaction formation.

B. repression. Repression is a defense mechanism that excludes unwanted or unpleasant experiences, emotions, or ideas from conscious awareness.

The major distinction between fear and anxiety is that fear: A. is a universal experience; anxiety is neurotic. B. enables constructive action; anxiety is dysfunctional. C. is a psychological experience; anxiety is a physiological experience. D. is a response to a specific danger; anxiety is a response to an unknown danger.

D. is a response to a specific danger; anxiety is a response to an unknown danger. Fear is a response to an objective danger; anxiety is a response to a subjective danger.

In the state of Oklahoma, an Emergency Order of Detention is valid for how long?

72 hours

A depressed patient says, "Nothing matters anymore." What is the most appropriate response by the nurse? a. "Are you having thoughts of suicide?" b. "I am not sure I understand what you are trying to say." c. "Try to stay hopeful. Things have a way of working out." d. "Tell me more about what interested you before you became depressed."

A) "Are you having thoughts of suicide?" The nurse must make overt what is covert; that is, the possibility of suicide must be openly addressed. The patient often feels relieved to be able to talk about suicidal ideation.

After one of their identical twin daughters commits suicide, the parents express concern that the other twin may also have suicidal tendencies. Which reply should the nurse provide? a. "Genetics are associated with suicide risk. Monitoring and support are important." b. "Apathy underlies suicide. Instilling motivation is the key to health maintenance." c. "Your child is unlikely to act out suicide when identifying with a suicide victim." d. "Fraternal twins are at higher risk for suicide than identical twins."

A) "Genetics are associated with suicide risk. Monitoring and support are important." Twin studies suggest the presence of genetic factors in suicide; however, separating genetic predisposition to suicide from predisposition to depression or alcoholism is difficult. Primary interventions can be helpful in promoting and maintaining health and possibly counteracting genetic load. The incorrect options are untrue statements or an oversimplification.

A patient says, "I've done a lot of cheating and manipulating in my relationships." Select a nonjudgmental response by the nurse. a. "How do you feel about that?" b. "I am glad that you realize this." c. "That's not a good way to behave." d. "Have you outgrown that type of behavior?"

A) "How do you feel about that?" Asking a patient to reflect on feelings about his or her actions does not imply any judgment about those actions, and it encourages the patient to explore feelings and values. The remaining options offer negative judgments.

Which comments by an adult best indicate self-actualization? (Select all that apply.) a. "I am content with a good book." b. "I often wonder if I chose the right career." c. "Sometimes I think about how my parents would have handled problems." d. "It's important for our country to provide basic health care services for everyone." e. "When I was lost at sea for 2 days, I gained an understanding of what is important."

A) "I am content with a good book." D) "It's important for our country to provide basic health care services for everyone." E) "When I was lost at sea for 2 days, I gained an understanding of what is important." Self-actualized persons enjoy privacy, have a sense of democracy, and show positive outcomes associated with peak experiences. Self-doubt, defensiveness, and blaming are not consistent with self-actualization.

Which comments by an elderly person best indicate successful completion of the individual's psychosocial developmental task? (Select all that apply.) a. "I am proud of my children's successes in life." b. "I should have given to community charities more often." c. "My relationship with my father made life more difficult for me." d. "My experiences in the war helped me appreciate the meaning of life." e. "I often wonder what would have happened if I had chosen a different career."

A) "I am proud of my children's successes in life." D) "My experiences in the war helped me appreciate the meaning of life." The developmental crisis for an elderly person relates to integrity versus despair. Pride in one's offspring indicates a sense of fulfillment. Recognition of the wisdom gained from difficult experiences (such as being in a war) indicates a sense of integrity. Blaming and regret indicate despair and unsuccessful resolution of the crisis.

A patient says, "Please don't share information about me with the other people." How should the nurse respond? a. "I will not share information with your family or friends without your permission, but I will share information about you with other staff." b. "A therapeutic relationship is just between the nurse and the patient. It is up to you to tell others what you want them to know." c. "It depends on what you choose to tell me. I will be glad to disclose at the end of each session what I will report to others." d. "I cannot tell anyone about you. It will be as though I am talking about my own problems, and we can help each other by keeping it between us."

A) "I will not share information with your family or friends without your permission, but I will share information about you with other staff." A patient has the right to know with whom the nurse will share information and that confidentiality will be protected. Although the relationship is primarily between the nurse and patient, other staff needs to know pertinent data. The other incorrect responses promote incomplete disclosure on the part of the patient, require daily renegotiation of an issue that should be resolved as the nurse-patient contract is established, and suggest mutual problem solving. The relationship must be patient centered. See relationship to audience response question.

To assess the client's perception of the event precipitating a crisis, the nurse would initially ask A. "What was happening just before you began feeling this way?" B. "During difficult times in the past, what has helped you?" C. "Can you give me the name of someone you trust?" D. "Who is available to help you?"

A) "What was happening just before you began feeling this way?" Option A is the only query that is directed at the client's perception of the precipitating event. The other options ask important questions but are not related to perception of the precipitating event.

Which findings are signs of a person who is mentally healthy? (Select all that apply.) a. Says, "I have some weaknesses, but I feel I'm important to my family and friends." b. Adheres strictly to religious beliefs of parents and family of origin. c. Spends all holidays alone watching old movies on television. d. Considers past experiences when deciding about the future. e. Experiences feelings of conflict related to changing jobs.

A) "Says, "I have some weaknesses, but I feel I'm important to my family and friends." D)Considers past experiences when deciding about the future. E)Experiences feelings of conflict related to changing jobs. Mental health is a state of well-being in which each individual is able to realize his or her own potential, cope with the normal stresses of life, work productively, and make a contribution to the community. Mental health provides people with the capacity for rational thinking, communication skills, learning, emotional growth, resilience, and self-esteem.

A person in the community asks, "People with mental illnesses went to state hospitals in earlier times. Why has that changed?" Select the nurse's accurate responses. (Select all that apply.) a. "Science has made significant improvements in drugs for mental illness, so now many persons may live in their communities." b. "There's now a better selection of less restrictive treatment options available in communities to care for people with mental illness." c. "National rates of mental illness have declined significantly. There actually is not a need for state institutions anymore." d. "Most psychiatric institutions were closed because of serious violations of patients' rights and unsafe conditions." e. "Federal legislation and payment for treatment of mental illness has shifted the focus to community rather than institutional settings."

A) "Science has made significant improvements in drugs for mental illness, so now many persons may live in their communities." B) "There's now a better selection of less restrictive treatment options available in communities to care for people with mental illness." E) "Federal legislation and payment for treatment of mental illness has shifted the focus to community rather than institutional settings." The community is a less restrictive alternative than hospitals for treatment of persons with mental illness. Funding for treatment of mental illness remains largely inadequate but now focuses on community rather than institutional care. Antipsychotic medications improve more symptoms of mental illness; hence, management of psychiatric disorders has improved. Rates of mental illness have increased, not decreased. Hospitals were closed because funding shifted to the community. Conditions in institutions have improved.

A patient repeatedly stated, "I'm stupid." Which statement by that patient would show progress resulting from cognitive-behavioral therapy? a. "Sometimes I do stupid things." b. "Things always go wrong for me." c. "I always fail when I try new things." d. "I'm disappointed in my lack of ability."

A) "Sometimes I do stupid things." "I'm stupid" is a cognitive distortion. A more rational thought is "Sometimes I do stupid things." The latter thinking promotes emotional self-control. The distracters reflect irrational or distorted thinking. This item relates to an audience response question.

A nurse makes an initial visit to a homebound patient diagnosed with a serious mental illness. A family member offers the nurse a cup of coffee. Select the nurse's best response. a. "Thank you. I would enjoy having a cup of coffee with you." b. "Thank you, but I would prefer to proceed with the assessment." c. "No, but thank you. I never accept drinks from patients or families." d. "Our agency policy prohibits me from eating or drinking in patients' homes."

A) "Thank you. I would enjoy having a cup of coffee with you." Accepting refreshments or chatting informally with the patient and family represent therapeutic use of self and help to establish rapport. The distracters fail to help establish rapport.

Carolina is surprised when her patient does not show for a regularly scheduled appointment. When contacted, the patient states, "I don't need to come see you anymore. I have found a therapy app on my phone that I love." How should Carolina respond to this news? a. "That sounds exciting, would you be willing to visit and show me the app?" b. "At this time, there is no real evidence that the app can replace our therapy." c. "I am not sure that is a good idea right now, we are so close to progress." d. "Why would you think that is a better option than meeting with me?"

A) "That sounds exciting, would you be willing to visit and show me the app?"

A nurse explains to the family of a mentally ill patient how a nurse-patient relationship differs from social relationships. Which is the best explanation? a. "The focus is on the patient. Problems are discussed by the nurse and patient, but solutions are implemented by the patient." b. "The focus shifts from nurse to patient as the relationship develops. Advice is given by both, and solutions are implemented." c. "The focus of the relationship is socialization. Mutual needs are met, and feelings are shared openly." d. "The focus is creation of a partnership in which each member is concerned with growth and satisfaction of the other."

A) "The focus is on the patient. Problems are discussed by the nurse and patient, but solutions are implemented by the patient." Only the correct response describes elements of a therapeutic relationship. The remaining responses describe events that occur in social or intimate relationships.

A patient states, "I'm starting cognitive-behavioral therapy. What can I expect from the sessions?" Which responses by the nurse would be appropriate? (Select all that apply.) a. "The therapist will be active and questioning." b. "You will be given some homework assignments." c. "The therapist will ask you to describe your dreams." d. "The therapist will help you look at your ideas and beliefs about yourself." e. "The goal is to increase subjectivity about thoughts that govern your behavior."

A) "The therapist will be active and questioning." B) "You will be given some homework assignments." D) "The therapist will help you look at your ideas and beliefs about yourself." Cognitive therapists are active rather than passive during therapy sessions because they help patient's reality-test their thinking. Homework assignments are given and completed outside the therapy sessions. Homework is usually discussed at the next therapy session. The goal of cognitive therapy is to assist the patient in identifying inaccurate cognitions and in reality-testing and formulating new, accurate cognitions. One distracter applies to psychoanalysis. Increasing subjectivity is not desirable.

A nurse in a mental health facility is caring for a client who has generalized anxiety disorder. Which of the following statements should the nurse make? A) "We'll assist you with making decisions" B) "Someone will work with you when you have flashbacks" C) "You'll be going through aversion therapy to help you cope" D) "The therapy will help control your impulses"

A) "We'll assist you with making decision" B: manifestations of PTSD, no anxiety

Which statement shows a nurse has empathy for a patient who made a suicide attempt? a. "You must have been very upset when you tried to hurt yourself." b. "It makes me sad to see you going through such a difficult experience." c. "If you tell me what is troubling you, I can help you solve your problems." d. "Suicide is a drastic solution to a problem that may not be such a serious matter."

A) "You must have been very upset when you tried to hurt yourself." Empathy permits the nurse to see an event from the patient's perspective, understand the patient's feelings, and communicate this to the patient. The incorrect responses are nurse-centered (focusing on the nurse's feelings rather than the patient's), belittling, and sympathetic.

A nurse assesses five newly hospitalized patients. Which patients have the highest suicide risk? (Select all that apply.) a. 82-year-old white male b. 17-year-old white female c. 22-year-old Hispanic male d. 19-year-old Native American male e. 39-year-old African American male

A) 82-year-old white male B) 17-year-old white female D) 19-year-old Native American male Whites have suicide rates almost twice those of nonwhites, and the rate is particularly high for older adult males, adolescents, and young adults. Other high-risk groups include youngAfrican American males, Native American males, and older Asian Americans. Rates are not high for Hispanic males.

Which statement about crisis theory will provide a basis for nursing intervention? a. A crisis is an acute, time-limited phenomenon experienced as an overwhelming emotional reaction to a problem perceived as unsolvable. b. A person in crisis has always had adjustment problems and has coped inadequately in his or her usual life situations. c. Crisis is precipitated by an event that enhances a person's self-concept and self-esteem. d. Nursing intervention in crisis situations rarely has the effect of ameliorating the crisis.

A) A crisis is an acute, time-limited phenomenon experienced as an overwhelming emotional reaction to a problem perceived as unsolvable.

A nurse surveys medical records. Which finding signals a violation of patients' rights? a. A patient was not allowed to have visitors. b. A patient's belongings were searched at admission. c. A patient with suicidal ideation was placed on continuous observation. d. Physical restraint was used after a patient was assaultive toward a staff member.

A) A patient was not allowed to have visitors. The patient has the right to have visitors. Inspecting patients' belongings is a safety measure. Patients have the right to a safe environment, including the right to be protected against impulses to harm self.

Which belief would be least helpful for a nurse working in crisis intervention? a. A person in crisis is incapable of making decisions. b. The crisis counseling relationship is one between partners. c. Crisis counseling helps the patient refocus to gain new perspectives on the situation. d. Anxiety-reduction techniques are used so the patient's inner resources can be accessed.

A) A person in crisis is incapable of making decisions.

Which patient is the best candidate for brief psychodynamic therapy? a. An accountant with a loving family and successful career who was involved in a short extramarital affair b. An adult with a long history of major depression who was charged with driving under the influence c. A woman with a history of borderline personality disorder who recently cut both wrists d. An adult male recently diagnosed with anorexia nervosa

A) An accountant with a loving family and successful career who was involved in a short extramarital affair The best candidates for psychodynamic therapy are relatively healthy and well-functioning individuals, sometimes referred to as the "worried well," who have a clearly circumscribed area of difficulty and are intelligent, psychologically minded, and well-motivated for change. Patients with psychosis, severe depression, borderline personality disorders, and severe character disorders are not appropriate candidates for this type of treatment.

Which statement provides the best rationale for closely monitoring a severely depressed patient during antidepressant medication therapy? a. As depression lifts, physical energy becomes available to carry out suicide. b. Patients who previously had suicidal thoughts need to discuss their feelings. c. For most patients, antidepressant medication results in increased suicidal thinking. d. Suicide is an impulsive act. Antidepressant medication does not alter impulsivity.

A) As depression lifts, physical energy becomes available to carry out suicide. Antidepressant medication has the objective of relieving depression. Risk for suicide is greater as the depression lifts, primarily because the patient has more physical energy at a time when he or she may still have suicidal ideation. The other options have little to do with nursing interventions relating to antidepressant medication therapy.

A community mental health nurse has worked with a patient for 3 years but is moving out of the city and terminates the relationship. When a novice nurse begins work with this patient, what is the starting point for the relationship? a. Begin at the orientation phase. b. Resume the working relationship. c. Initially establish a social relationship. d. Return to the emotional catharsis phase.

A) Begin at the orientation phase. After termination of a long-term relationship, the patient and new nurse usually have to begin at ground zero, the orientation phase, to build a new relationship. If termination is successfully completed, the orientation phase sometimes progresses quickly to the working phase. Other times, even after successful termination, the orientation phase may be prolonged.

When should a nurse be most alert to the possibility of communication errors resulting in harm to the patient? a. Change of shift report b. Admission interviews c. One-to-one conversations with patients d. Conversations with patient families

A) Change of shift report

The health care team at an inpatient psychiatric facility drafts these criteria for admission. Which criteria should be included in the final version of the admission policy? (Select all that apply) a. Clear risk of danger to self or others b. Adjustment needed for doses of psychotropic medication c. Detoxification from long-term heavy alcohol consumption needed d. Respite for caregivers of persons with serious and persistent mental illness e. Failure of community-based treatment, demonstrating need

A) Clear risk of danger to self or others C) Detoxification from long-term heavy alcohol consumption needed E)Failure of community-based treatment, demonstrating need Medication doses can be adjusted on an outpatient basis. The goal of caregiver respite can be accomplished without hospitalizing the patient. The other options are acceptable, evidence-based criteria for admission of a patient to an inpatient service.

At what point in the nurse-patient relationship should a nurse plan to first address termination? a. During the orientation phase b. At the end of the working phase c. Near the beginning of the termination phase d. When the patient initially brings up the topic

A) During the orientation phase The patient has a right to know the conditions of the nurse-patient relationship. If the relationship is to be time-limited, the patient should be informed of the number of sessions. If it is open-ended, the termination date will not be known at the outset, and the patient should know that the issue will be negotiated at a later date. The nurse is responsible for bringing up the topic of termination early in the relationship, usually during the orientation phase.

A nurse ends a relationship with a patient. Which actions by the nurse should be included in the termination phase? (Select all that apply.) a. Focus dialogues with the patient on problems that may occur in the future. b. Help the patient express feelings about the relationship with the nurse. c. Help the patient prioritize and modify socially unacceptable behaviors. d. Reinforce expectations regarding the parameters of the relationship. e. Help the patient to identify strengths, limitations, and problems.

A) Focus dialogues with the patient on problems that may occur in the future. B) Help the patient express feelings about the relationship with the nurse. The correct actions are part of the termination phase. The other actions would be used in the working and orientation phases.

Select the example of tertiary prevention. a. Helping a person diagnosed with a serious mental illness learn to manage money b. Restraining an agitated patient who has become aggressive and assaultive c. Teaching school-age children about the dangers of drugs and alcohol d. Genetic counseling with a young couple expecting their first child

A) Helping a person diagnosed with a serious mental illness learn to manage money Tertiary prevention involves services that address residual impairments, with a goal of improved independent functioning. Restraint is a secondary prevention. Genetic counseling and teaching school-age children about substance abuse and dependence are examples of primary prevention.

A 4-year-old grabs toys from other children and says, "I want that now!" From a psychoanalytic perspective, this behavior is a product of impulses originating in which system of the personality? a. Id b. Ego c. Superego d. Preconscious

A) ID The id operates on the pleasure principle, seeking immediate gratification of impulses. The ego acts as a mediator of behavior and weighs the consequences of the action, perhaps determining that taking the toy is not worth the mother's wrath. The superego would oppose the impulsive behavior as "not nice." The preconscious is a level of awareness rather than an aspect of personality.

A nurse is establishing a therapeutic relationship with a client who hallucinates. Which of the following actions should the nurse take during the orientation phase? A) Identify the client's perception of the reason for therapy B) Ask the client to provide a detailed description of the hallucinations C) Assist the client with the development of problem-solving skills D) Explore the client's relationship with family members

A) Identify the client's perception of the reason for therapy All other options are apart of the working phase! Talk about the termination phase throughout all 3 beginning phases

Which nursing interventions will be implemented for a patient who is actively suicidal?(Select all that apply.) a. Maintain arm's length, one-on-one direct observation at all times. b. Check all items brought by visitors and remove risk items. c. Use plastic eating utensils; count utensils upon collection. d. Remove the patient's eyeglasses to prevent self-injury. e. Interact with the patient every 15 minutes.

A) Maintain arm's length, one-on-one direct observation at all times B) Check all items brought by visitors and remove risk items C) Use plastic eating utensils; count utensils upon collection. One-on-one observation is necessary for anyone who has limited or unreliable control over suicidal impulses. Finger foods allow the patient to eat without silverware; "no silver or glassware" orders restrict access to a potential means of self-harm. Every-15-minute checks are inadequate to assure the safety of an actively suicidal person. Placement in a public area is not a substitute for arm's-length direct observation; some patients will attempt suicide even when others are nearby. Vision impairment requires eyeglasses (or contacts); although they could be used dangerously, watching the patient from arm's length at all times would allow enough time to interrupt such an attempt and would prevent the disorientation and isolation that uncorrected visual impairment could create.

Which situation has the potential for early crisis intervention to occur? A. Mrs. R tells the nurse in the well-baby clinic that she's feeling uptight and has arranged to see a primary care therapist. B. Ms. T is hospitalized after an unsuccessful suicide attempt that she states, "was a mistake." C. Mr. W asks for reassurance that he will be welcome at the day hospital after his hospital discharge. D. Ms. G enters the emergency department with a strong smell of alcohol on his person, stating he is anxious and depressed.

A) Mrs. R tells the nurse in the well-baby clinic that she's feeling uptight and has arranged to see a primary care therapist. Phase I intervention is when a person confronted by a conflict or problem that threatens the self-concept responds with increased feelings of anxiety. The increase in anxiety stimulates the use of problem-solving techniques and defense mechanisms in an effort to solve the problem and lower anxiety. Option B indicates a phase 4 response to a crisis; option C would be a phase 3 response. In option D the client is using inappropriate coping mechanisms, which are not effective to treat depression and anxiety.

A patient is suspicious and frequently manipulates others. To which psychosexual stage do these traits relate? a. Oral b. Anal c. Phallic d. Genital

A) Oral The behaviors in the stem develop as the result of attitudes formed during the oral stage, when an infant first learns to relate to the environment. Anal-stage traits include stinginess, stubbornness, orderliness, or their opposites. Phallic-stage traits include flirtatiousness, pride, vanity, difficulty with authority figures, and difficulties with sexual identity. Genital-stage traits include the ability to form satisfying sexual and emotional relationships with members of the opposite sex, emancipation from parents, a strong sense of personal identity, or the opposites of these traits.

What principle about nurse-patient communication should guide a nurse's fear about "saying the wrong thing" to a patient? a. Patients tend to appreciate a well-meaning person who conveys genuine acceptance, respect, and concern for their situation. b. The patient is more interested in talking to you than listening to what you have to say and so is not likely to be offended. c. Considering the patient's history, there is little chance that the comment will do any actual harm. d. Most people with a mentally illness have by necessity developed a high tolerance of forgiveness.

A) Patients tend to appreciate a well-meaning person who conveys genuine acceptance, respect, and concern for their situation.

A nurse performed these actions while caring for patients in an inpatient psychiatric setting. Which action violated patients' rights? a. Prohibited a patient from using the telephone b. In patient's presence, opened a package mailed to patient c. Remained within arm's length of patient with homicidal ideation d. Permitted a patient with psychosis to refuse oral psychotropic medication

A) Prohibited a patient from using the telephone The patient has a right to use the telephone. The patient should be protected against possible harm to self or others. Patients have rights to send and receive mail and be present during package inspection. Patients have rights to refuse treatment.

A nurse in a mental health unit is planning care for a client who is receiving treatment for self-inflicted injuries. The nurse should identify which of the following interventions as the priority when planning care for this client? A) Promoting and maintaining client safety B) Discussing reasons for the client's behavior C) Assisting the client in recognizing feelings of self-harm D) Teaching the client alternative coping strategies

A) Promoting and maintaining client safety

Which technique is most applicable to aversion therapy? a. Punishment b. Desensitization c. Role modeling d. Positive reinforcement

A) Punishment Aversion therapy is akin to punishment. Aversive techniques include pairing of a maladaptive behavior with a noxious stimulus, punishment, and avoidance training.

A nurse is assessing a newly admitted client who has generalized anxiety disorder and states, "I drink alcohol to forget the pain." The client is exhibiting a maladaptive response related to which of the following defense mechanisms? A) Rationalization B) Conversion C) Projection D) Suppression

A) Rationalization

A patient diagnosed with schizophrenia lives in the community. On a home visit, the community psychiatric nurse case manager learns that the patient:• wants to attend an activity group at the mental health outreach center.• is worried about being able to pay for the therapy.• does not know how to get from home to the outreach center.• has an appointment to have blood work at the same time an activity group meets.• wants to attend services at a church that is a half-mile from the patient's home.Which tasks are part of the role of a community mental health nurse? (Select all that apply.) a. Rearranging conflicting care appointments b. Negotiating the cost of therapy for the patient c. Arranging transportation to the outreach center d. Accompanying the patient to church services weekly e. Monitoring to ensure the patient's basic needs are met

A) Rearranging conflicting care appointments C) Arranging transportation to the outreach center E)Monitoring to ensure the patient's basic needs are met The correct answers reflect the coordinating role of the community psychiatric nurse case manager. Negotiating the cost of therapy and accompanying the patient to church services are interventions the nurse would not be expected to undertake. The patient can walk to the church services; the nurse can provide encouragement.

As a nurse discharges a patient, the patient gives the nurse a card of appreciation made in an arts and crafts group. What is the nurse's best action? a. Recognize the effectiveness of the relationship and patient's thoughtfulness. Accept the card. b. Inform the patient that accepting gifts violates policies of the facility. Decline the card. c. Acknowledge the patient's transition through the termination phase but decline the card. d. Accept the card and invite the patient to return to participate in other arts and crafts groups.

A) Recognize the effectiveness of the relationship and patient's thoughtfulness. Accept the card. The nurse must consider the meaning, timing, and value of the gift. In this instance, the nurse should accept the patient's expression of gratitude. See relationship to audience response question.

A nurse uses Maslow's hierarchy of needs to plan care for a patient diagnosed with mental illness. Which problem will receive priority? The patient a. refuses to eat or bathe. b. reports feelings of alienation from family. c. is reluctant to participate in unit social activities. d. is unaware of medication action and side effects.

A) Refuses to eat or bathe The need for food and hygiene are physiological and therefore take priority over psychological or meta-needs in care planning.

Which principle has the highest priority when addressing a behavioral crisis in an inpatient setting? a. Resolve the crisis with the least restrictive intervention possible. b. Swift intervention is justified to maintain the integrity of a therapeutic milieu. c. Rights of an individual patient are superseded by the rights of the majority of patients. d. Patients should have opportunities to regain control without intervention if the safety of others is not compromised.

A) Resolve the crisis with the least restrictive intervention possible. The rule of using the least restrictive treatment or intervention possible to achieve the desired outcome is the patient's legal right. Planned interventions are nearly always preferable. Intervention may be necessary when the patient threatens harm to self.

A nurse says, "I am the only one who truly understands this patient. Other staff members are too critical." The nurse's statement indicates a. boundary blurring. b. sexual harassment. c. positive regard. d. advocacy.

A) boundary blurring. When the role of the nurse and the role of the patient shift, boundary blurring may arise. In this situation the nurse is becoming overinvolved with the patient as a probable result of unrecognized countertransference. When boundary issues occur, the need for supervision exists. The situation does not describe sexual harassment. Data are not present to suggest positive regard or advocacy.

The priority concern of the crisis intervention nurse is A. client safety. B. setting up future contacts. C. brainstorming possible solutions. D. working through termination issues.

A) client safety Client safety is always the priority concern in crisis intervention therapy. The disequilibrium of crisis predisposes the client to suicidal thinking.

A novice psychiatric nurse has a parent diagnosed with bipolar disorder. This nurse angrily recalls feelings of embarrassment about the parent's behavior in the community. Select the best ways for this nurse to cope with these feelings. (Select all that apply.) a. Seek ways to use the understanding gained from childhood to help patients cope with their own illnesses. b. Recognize that these feelings are unhealthy. The nurse should try to suppress them when working with patients. c. Recognize that psychiatric nursing is not an appropriate career choice. Explore other nursing specialties. d. The nurse should begin new patient relationships by saying, "My own parent had mental illness, so I accept it without stigma." e. Recognize that the feelings may add sensitivity to the nurse's practice, but supervision is important.

A) Seek ways to use the understanding gained from childhood to help patients cope with their own illnesses. E) Recognize that the feelings may add sensitivity to the nurse's practice, but supervision is important. The nurse needs support to explore these feelings. An experienced psychiatric nurse is a resource that may be helpful. The knowledge and experience gained from the nurse's relationship with a mentally ill parent may contribute sensitivity to compassionate practice. Self-disclosure and suppression are not adaptive coping strategies. The nurse should not give up on this area of practice without first seeking ways to cope with the memories.

Which finding best indicates that the goal "Demonstrate mentally healthy behavior" was achieved for an adult patient? The patient a) sees self as capable of achieving ideals and meeting demands. b) behaves without considering the consequences of personal actions. c) aggressively meets own needs without considering the rights of others. d) seeks help from others when assuming responsibility for major areas of own life.

A) Sees self as capable of achieving ideals and meeting demands The correct response describes an adaptive, healthy behavior. The distracters describe maladaptive behaviors.

It has been 5 days since a suicidal patient was hospitalized and prescribed an antidepressant medication. The patient is now more talkative and shows increased energy. Select the highest priority nursing intervention. a. Supervise the patient 24 hours a day. b. Begin discharge planning for the patient. c. Refer the patient to art and music therapists. d. Consider discontinuation of suicide precautions.

A) Supervise the patient 24 hours a day. The patient now has more energy and may have decided on suicide, especially given the prior suicide attempt history. The patient must be supervised 24 hours per day. The patient is still a suicide risk.

Which assumption serves as a foundation for the use of crisis intervention? A. The individual is mentally healthy but in a state of disequilibrium. B. Long-term dysfunctional adjustment can be addressed by crisis intervention. C. An anxious person is unlikely to be willing to try new problem-solving strategies. D. Crisis intervention nurses need to remain passive as the client deals with the crisis.

A) The individual is mentally healthy but in a state of disequilibrium

A patient diagnosed with schizophrenia has been stable for 2 months. Today the patient's spouse calls the nurse to report the patient has not taken prescribed medication and is having disorganized thinking. The patient forgot to refill the prescription. The nurse arranges a refill. Select the best outcome to add to the plan of care. a. The patient's spouse will mark dates for prescription refills on the family calendar. b. The nurse will obtain prescription refills every 90 days and deliver to the patient. c. The patient will call the nurse weekly to discuss medication-related issues. d. The patient will report to the clinic for medication follow-up every week.

A) The patient's spouse will mark dates for prescription refills on the family calendar. The nurse should use the patient's support system to meet patient needs whenever possible. Delivery of medication by the nurse should be unnecessary for the nurse to do if patient or a significant other can be responsible. The patient may not need more intensive follow-up as long as medication is taken as prescribed.

Consider this comment from a therapist: "The patient is homosexual but has kept this preference secret. Severe anxiety and depression occur when the patient anticipates family reactions to this sexual orientation." Which perspective is evident in the speaker? a. Theory of interpersonal relationships b. Classical conditioning theory c. Psychosexual theory d. Behaviorism theory

A) Theory of interpersonal relationships The theory of interpersonal relationships recognizes the anxiety and depression as resulting from unmet interpersonal security needs. Behaviorism and classical conditioning theories do not apply. A psychosexual formulation would focus on uncovering unconscious material that relates to the patient problem.

A community mental health nurse has worked for months to establish a relationship with a delusional, suspicious patient. The patient recently lost employment and could no longer afford prescribed medications. The patient says, "Only a traitor would make me go to the hospital." Select the nurse's best initial intervention. a. With the patient's consent, contact resources to provide medications without charge temporarily. b. Arrange a bed in a local homeless shelter with nightly on-site supervision. c. Hospitalize the patient until the symptoms have stabilized. d. Ask the patient, "Do you feel like I am a traitor?"

A) With the patient's consent, contact resources to provide medications without charge temporarily. Hospitalization may damage the nurse-patient relationship, even if it provides an opportunity for rapid stabilization. If medication is restarted, the patient may possibly be stabilized in the home setting, even if it takes a little longer. Programs are available to help patients who are unable to afford their medications. A homeless shelter is inappropriate and unnecessary. Hospitalization may be necessary later, but a less restrictive solution should be tried first, since the patient is not dangerous. A yes/no question is non-therapeutic communication.

During an admission assessment and interview, which channels of information communication should the nurse be monitoring? Select all that apply. a. Auditory b. Visual c. Written d. Tactile e. Olfactory

A) auditory B) visual D) tactile E)olfactory

When a stressful event occurs and the individual is unable to resolve the situation by using his or her usual coping strategies, the individual A. becomes disorganized and uses trial-and-error problem solving. B. withdraws and acts as though the problem does not exist. C. develops severe personality disorganization. D. resorts to planning suicide

A) becomes disorganized and uses trial-and-error problem solving. This is the second stage of crisis, according to accepted crisis theory.

In the majority culture of the United States, which individual has the greatest risk to be labeled mentally ill? One who a. describes hearing God's voice speaking. b. is usually pessimistic but strives to meet personal goals c. is wealthy and gives away $20 bills to needy individuals. d. always has an optimistic viewpoint about life and having own needs met.

A) describes hearing God's voice speaking The question asks about risk. Hearing voices is generally associated with mental illness, but in charismatic religious groups, hearing the voice of God or a prophet is a desirable event. Cultural norms vary, which makes it more difficult to make an accurate diagnosis. The individuals described in the other options are less likely to be labeled mentally ill.

A client nervously says, "financial problems are stressing my marriage. I've heard rumors about cutbacks at work and I am afraid I might get laid off." The client's pulse is 112 beats/minute; respirations are 26 breaths/minute ; and blood pressure is 166/88. Which nursing intervention will the nurse implement? A) Direct the client in slow and deep breathing using abdominal muscles B) Advise the client, "go to sleep to 30 to 60 minutes earlier each night to increase rest C) Tell the client, "Relax by spending more time playing with your pet." D) Suggest the client consider that a new job might be better than the present one

A) direct the client in slow and deep breathing using abdominal muscles

A college student who attempted suicide by overdose was hospitalized. When the parentswere contacted, they responded, "We should have seen this coming. We did not do enough."The parents' reaction reflects a. guilt. b. denial. c. shame. d. rescue feelings.

A) guilt The parents' statements indicate guilt. Guilt is evident from the parents' self-chastisement.The feelings suggested in the distracters are not clearly described in the scenario.

A psychiatric nurse discusses rules of the therapeutic milieu and patients' rights with a newly admitted patient. Which rights should be included? The right to (Select all that apply) a. have visitors. b. confidentiality. c. a private room. d. complain about inadequate care. e. select the nurse assigned to their care.

A) have visitors B) confidentiality D) Complain about inadequate care Patients' rights should be discussed shortly after admission. Patients have rights related to receiving/refusing visitors, privacy, filing complaints about inadequate care, and accepting/refusing treatments (including medications). Patients do not have a right to a private room or selecting which nurse will provide care.

The feeling experienced by a patient that should be assessed by the nurse as most predictive of elevated suicide risk is a. hopelessness. b. sadness. c. elation. d. anger.

A) hopelessness Of the feelings listed, hopelessness is most closely associated with increased suicide risk.Depression, aggression, impulsivity, and shame are other feelings noted as risk factors for suicide.

A nurse can best address factors of critical importance to successful community treatment by including making assessments relative to (Select all that apply) a. housing adequacy. b. family and support systems. c. income adequacy and stability. d. early psychosocial development. e. substance abuse history and current use.

A) housing adequacy B) Family and support system C) Income adequacy and stability E) Substance abuse history and current use Early psychosocial developmental history is less relevant to successful outcomes in the community than the assessments listed in the other options. If a patient is homeless or fears homelessness, focusing on other treatment issues is impossible. Sufficient income for basic needs and medication is necessary. Adequate support is a requisite to community placement. Substance abuse undermines medication effectiveness and interferes with community adjustment.

The highest-priority goal of crisis intervention is: a. Patient safety. b. Anxiety reduction. c. Identification of situational supports. d. Teaching specific coping skills that are lacking.

A) patient safety

Mason and Charlie, both 16 years old, were involved in a bad car accident in which they were both passengers. Mason spoke with a counselor about the incident once and has been able to move forward with little dysfunction. Charlie has been experiencing anxiety and an inability to concentrate in school even after numerous counseling sessions. The difference in the way the accident affected both boys may be explained by: A. perception of the event. B. Mason's more laid-back personality. C. the possibility that Charlie may have experienced previous trauma from which he did not fully recover. D. the possibility that counseling Charlie received may have been inadequate.

A) perception of the event. People vary in the way they absorb, process, and use information from the environment. Some people may respond to a minor event as if it were life-threatening. Conversely, others may experience a major event and look at it in a calmer fashion. The other options may be true but are not the primary reason two people respond differently to the same event.

Inpatient hospitalization for persons with mental illness is generally reserved for patients who a. present a clear danger to self or others. b. are noncompliant with medication at home. c. have limited support systems in the community. d. develop new symptoms during the course of an illness

A) present a clear danger to self or others. Hospitalization is justified when the patient is a danger to self or others, has dangerously decompensated, or needs intensive medical treatment. The distracters do not necessarily describe patients who require inpatient treatment

When responding to the patient in question 2, the intervention that takes priority is to: a. Reduce anxiety. b. Arrange shelter. c. Contact out-of-area family. D. Hospitalize and place the patient on suicide precautions

A) reduce anxiety

A college student is extremely upset after failing two examinations. The student said, "No one understands how this will hurt my chances of getting into medical school." The student then suspends access to his social networking website and turns off his cell phone. Which suicide risk factors are evident? (Select all that apply.) a. Shame b. Panic attack c. Humiliation d. Self-imposed isolation e. Recent stressful life event

A) shame C) humiliation D) self-imposed isolation E) recent stressful life event Failing examinations in the academic major constitutes a recent stressful life event. Shame and humiliation related to the failure can be hypothesized. The statement, "No one can understand," can be seen as recent lack of social support. Terminating access to one's social networking site and turning off the cell phone represents self-imposed isolation. The scenario does not provide evidence of panic attack.

Which statements by patients diagnosed with a serious mental illness best demonstrate that the case manager has established an effective long-term relationship? "My case manager (Select all that apply) a. talks in language I can understand." b. helps me keep track of my medication." c. gives me little gifts from time to time." d. looks at me as a whole person with many needs." e. let me do whatever I choose without interfering."

A) talks in language I can understand." B) helps me keep track of my medication." D) looks at me as a whole person with many needs." Each correct answer is an example of appropriate nursing foci: communicating at a level understandable to the patient, providing medication supervision, and using holistic principles to guide care. The distracters violate relationship boundaries or suggest a laissez faire attitude on the part of the nurse.

Which scenario best depicts a behavioral crisis? A patient is a. waving fists, cursing, and shouting threats at a nurse. b. curled up in a corner of the bathroom, wrapped in a towel. c. crying hysterically after receiving a phone call from a family member. d. performing push-ups in the middle of the hall, forcing others to walk around.

A) waving fists, cursing, and shouting threats at a nurse. This behavior constitutes a behavioral crisis because the patient is threatening harm to another individual. Intervention is called for to defuse the situation. The other options speak of behaviors that may require intervention of a less urgent nature because the patients in question are not threatening harm to self or others.

A nurse is caring for a client who is failing one of their school courses. The client blames the teacher for their bad grade. Which of the following defense mechanisms is the client exhibiting? A) Projection B) Dissociation C) Undoing D) Compensation

A)Projection

The psychiatric nurse planning and implementing care for culturally diverse clients should understand A. holistic theory. B. systems theory. C. adaptation theory. D. political power theory.

A. Holistic theory In most cultures a holistic perspective prevails, one without separation of mind and body.

The nurse assesses the wellness beliefs and values of a client from another culture best when asking A. "What do you think is making you ill?" B. "When did you first feel ill?" C. "How can I help you get better?" D. "Did you do something to cause the illness?"

A. "What do you think is making you ill?" Asking the client to suggest reasons for the illness will best provide an opportunity to become familiar with general beliefs and values the client holds regarding his wellness.

A client who is demonstrating a moderate level of anxiety tells the nurse, "I am so anxious, and I do not know what to do." A helpful response for the nurse to make would be: A. "What things have you done in the past that helped you feel more comfortable?" B. "Let's try to focus on that adorable little granddaughter of yours." C. "Why don't you sit down over there and work on that jigsaw puzzle?" D. "Try not to think about the feelings and sensations you're experiencing."

A. "What things have you done in the past that helped you feel more comfortable?" Because the client is not able to think through the problem and arrive at an action that would lower anxiety, the nurse can assist by asking what has worked in the past. Often what has been helpful in the past can be used again.

Which of the following patients may be an appropriate candidate for a release from hospitalization known as against medical advice (AMA)? A. 37-year-old patient hospitalized for 6 days; the provider feels one more day would benefit the patient, but the patient doesn't agree and wishes to be discharged B. 75-year-old patient with dementia who demands to be allowed to go back to his own home C. 21-year-old actively suicidal patient on the psychiatric unit who wants to be discharged to home and do outpatient counseling D. 32-year-old female patient who wishes to stay in the hospital but whose husband demands that she be discharged into his care

A. 37-year-old patient hospitalized for 6 days; the provider feels one more day would benefit the patient, but the patient doesn't agree and wishes to be discharged Rationale: Against medical advice discharges are sometimes used when the patient does not agree with the provider, as long as the patient is not a danger to himself or herself or to others. The patient with dementia and the patient who is actively suicidal would pose a safety risk and would be not allowed to be discharged AMA. A patient who wishes to stay in the hospital can make that decision; a family member's opinion doesn't impact an AMA discharge.

Jonas is a 29-year-old patient with anxiety and a history of alcohol abuse who is an inpatient on the psychiatric unit. He becomes angry and aggressive, strikes another patient, and then attacks a staff member. He is taken to seclusion and medicated with haloperidol and lorazepam. In this case, the haloperidol and lorazepam may be considered: A. A restraint. B. A medication time-out. C. False imprisonment. D. Malpractice.

A. A restraint. Rationale: Chemical restraints are defined by those medications or doses of medication that are not being used for the patient's condition. Medication time-out is incorrect; false imprisonment and malpractice refer to specific legal terms that do not have any bearing on this medication scenario.

The use of seclusion or restraint to control the behavior of a client who is at risk of harming self or others gives rise to conflict between the ethical principles of: A. Autonomy and beneficence. B. Advocacy and confidentiality. C. Veracity and fidelity. D. Justice and humanism.

A. Autonomy and beneficence. Rationale: Autonomy refers to self-determination and beneficence refers to doing good. When a client is restrained or secluded, the need to do good and prevent harm outweighs the client's autonomy.

A nurse is adequately representing the stated bioethical principle when valuing: A. Autonomy by respecting a client's right to decide to refuse cancer treatment. B. Justice by staying with a client who is suicidal. C. Fidelity by informing the client about the negative side effects of a proposed treatment. D. Beneficence when advocating for a client's right to enter into a clinical trial for a new medication.

A. Autonomy by respecting a client's right to decide to refuse cancer treatment. Rationale: Autonomy refers to self-determination. Self-determination can be exercised when one makes his or her own decisions without interference from others.

Which statement best explains the term "worldview"? A. Beliefs and values held by people of a given culture about what is good, right, and normal. B. Ideas derived from the major health care system of the culture about what causes illness. C. Cultural norms about how, when, and to whom illness symptoms may be displayed. D. Valuing one's beliefs and customs over those of another group.

A. Beliefs and values held by people of a given culture about what is good, right, and normal. A worldview is a system of thinking about how the world works and how people should behave in the world and toward each other. It is from this view that people develop beliefs, values, and the practices that guide their lives.

A possible outcome criterion for a client diagnosed with anxiety disorder is: A. Client demonstrates effective coping strategies. B. Client reports reduced hallucinations. C. Client reports feelings of tension and fatigue. D. Client demonstrates persistent avoidance behaviors.

A. Client demonstrates effective coping strategies. Option A is the only desirable outcome listed.

A client is released from involuntary commitment by the judge, who orders that a caseworker supervise him for the next 6 months. This is an example of: A. Conditional discharge. B. Outpatient commitment. C. Voluntary follow-up. D. Discretionary treatment.

A. Conditional discharge. Rationale: An unconditional discharge gives the client complete freedom to choose or reject follow-up care. A conditional discharge imposes a legal requirement for the client to submit to follow-up supervision.

A nurse administering a benzodiazepine should understand that the therapeutic effect of benzodiazepines results from potentiating the neurotransmitter: A. GABA B. Dopamine C. Serotonin D. Acetylcholine E. A and C

A. GABA

Which statement concerning the right to treatment in public psychiatric hospitals is accurate? A. Hospitalization without treatment violates the client's rights. B. Right to treatment extends only to provision of food, shelter, and safety. C. All clients have the right to choose a primary therapist and case manager. D. The right to treatment for hallucinations has priority over treatment for anxiety.

A. Hospitalization without treatment violates the client's rights. Rationale: Many years ago psychiatric clients were warehoused in large mental institutions, given custodial care, and rarely released into the community. As enlightenment occurred, it was determined that each client who is hospitalized has the right to receive treatment.

The charge nurse shares with the psychiatric technician that negligence of a patient A. Is an act or failure to act in a way that a responsible employee would act. B. Applies only when the client is abandoned or mistreated. C. Is an action that puts the client in fear of being harmed by the employee. D. Means the employee has given malicious false information about the client.

A. Is an act or failure to act in a way that a responsible employee would act. Rationale: Behaving as a wise and prudent person would act under the same circumstances is one way of judging whether the standard of care has been violated. Employers typically hope that staff will prevent clients from striking each other.

In the course of providing best psychiatric care for a client, the nurse must place greatest reliance on A. Legal principles. B. Ethical principles. C. Independent judgment. D. Institutional standards.

A. Legal principles. Rationale: Legal principles are fundamental to nursing practice. They supersede all other principles, standards, and judgments. All students are encouraged to become familiar with the important provisions of the laws in their own states regarding admissions, discharges, clients' rights, and informed consent.

The nurse administers each of the following drugs to various patients. The patient who should be most carefully assessed for fluid and electrolyte imbalance is the one receiving: A. Lithium B. Clozapine C. Diazepam D. Amitriptyline

A. Lithium

The civil rights of persons with mental illness who are hospitalized for treatment are: A. The same as those for any other citizen. B. Altered to prevent use of poor judgment. C. Always ensured by appointment of a guardian. D. Limited to provision of humane treatment.

A. The same as those for any other citizen. Rationale: Civil rights are not lost because of hospitalization for mental illness.

Which idea held by the nurse would best promote the provision of culturally competent care? A. Western biomedicine is one of several established healing systems. B. Some individuals will profit from use of both Western and folk healing practices. C. Use of cultural translators will provide valuable information into health-seeking behaviors. D. Need for spiritual healing is a concept that crosses cultural boundaries.

A. Western biomedicine is one of several established healing systems. A nurse who holds this belief would be likely be open to a variety of established interventions. In truth, nurses cannot apply a standard model of assessment, diagnosis, and intervention to all clients with equal confidence. This leads to culturally irrelevant interventions.

In the Eastern tradition, disease is believed to be caused by A. fluctuations in opposing forces. B. outside influences. C. members' disobedience. D. adoption of Western beliefs.

A. fluctuations in opposing forces. In the Eastern tradition, disease is believed to be caused by fluctuations in opposing forces, the yin-yang energies.

Generally, ego defense mechanisms: A. often involve some degree of self-deception. B. are rarely used by mentally healthy people. C. seldom make the person more comfortable. D. are usually effective in resolving conflicts.

A. often involve some degree of self-deception. Most ego defense mechanisms, with the exception of the mature defenses, alter the individual's perception of reality to produce varying degrees of self-deception.

Inability to leave one's home because of avoidance of severe anxiety suggests the anxiety disorder of: A. panic attacks with agoraphobia. B. obsessive-compulsive disorder. C. posttraumatic stress response. D. generalized anxiety disorder.

A. panic attacks with agoraphobia. Panic disorder with agoraphobia is characterized by recurrent panic attacks combined with agoraphobia. Agoraphobia involves intense, excessive anxiety about being in places or situations from which escape might be difficult or embarrassing or in which help might not be available if a panic attack occurred.

This term means acknowledging a patient's right to make their own health care decisions

Autonomy

Which statement by a patient would lead the nurse to suspect unsuccessful completion of the psychosocial developmental task of infancy? a. "I know how to do things right, so I prefer jobs where I work alone rather than on a team." b. "I do not allow other people to truly get to know me." c. "I depend on frequent praise from others to feel good about myself." d. "I usually need to do things several times before I get them right."

B) "I do not allow other people to truly get to know me." According to Erikson, the developmental task of infancy is the development of trust. The correct response is the only statement clearly showing lack of ability to trust others. An inability to work with others, coupled with a sense of superiority, suggests unsuccessful completion of the task of intimacy versus isolation. Relying on praise from others suggests unsuccessful completion of the task of identity versus role confusion. Shame suggests failure to resolve the crisis of initiative versus guilt.

Which statement by a depressed patient will alert the nurse to the patient's need for immediate, active intervention? a. "I am mixed up, but I know I need help." b. "I have no one to turn to for help or support." c. "It is worse when you are a person of color." d. "I tried to get attention before I cut myself last time."

B) "I have no one to turn to for help or support." Hopelessness is evident. Lack of social support and social isolation increases the suicide risk.Willingness to seek help lowers risk. Being a person of color does not suggest higher risk because more whites commit suicide than do individuals of other racial groups. Attention seeking is not correlated with higher suicide risk.

Which statement would suggest to the crisis intervention nurse the need to arrange for hospitalization of a client? A. "I'm feeling overwhelmed by all that has happened, and I need help sorting it out." B. "I see no solution for this situation if nothing changes by tomorrow." C. "There are three possibilities that might help, but I can't decide what to do." D. "I feel a little calmer than yesterday at this time, but things are still very difficult."

B) "I see no solution for this situation if nothing changes by tomorrow." Whenever the client presents a danger to himself or herself or others, hospitalization must be considered.

A nurse is caring for a client who has depression. The nurse observes that the client has not come to breakfast and is still in bed. The client states, "I am not worth your time. Leave me alone and go help someone else." Which of the following responses should the nurse make? A) "Many people feel this way when they first start treatment" B) "In other words, you seem to be saying you feel unworthy of help" C) "You'll feel better once you get up and have some breakfast" D) "I disagree with your feeling that you are not worthy of my time"

B) "In other words, you seem to be saying you feel unworthy of help" A: don't generalize/invalidate C: don't make false promises D: Don't outright disagree with patients

An advanced practice nurse observes a novice nurse expressing irritability regarding a patient with a long history of alcoholism and suspects the new nurse is experiencing countertransference. Which comment by the new nurse confirms this suspicion? a. "This patient continues to deny problems resulting from drinking." b. "My parents were alcoholics and often neglected our family." c. "The patient cannot identify any goals for improvement." d. "The patient said I have many traits like her mother."

B) "My parents were alcoholics and often neglected our family." Countertransference occurs when the nurse unconsciously and inappropriately displaces onto the patient feelings and behaviors related to significant figures in the nurse's past. In this instance, the new nurse's irritability stems from relationships with parents. The distracters indicate transference or accurate analysis of the patient's behavior.

A student nurse says, "I don't need to interact with my patients. I learn what I need to know by observation." An instructor can best interpret the nursing implications of Sullivan's theory to this student by responding: a. "Interactions are required in order to help you develop therapeutic communication skills." b. "Nurses cannot be isolated. We must interact to provide patients with opportunities to practice interpersonal skills." c. "Observing patient interactions will help you formulate priority nursing diagnoses and appropriate interventions." d. "It is important to pay attention to patients' behavioral changes, because these signify adjustments in personality."

B) "Nurses cannot be isolated. We must interact to provide patients with opportunities to practice interpersonal skills." The nurse's role includes educating patients and assisting them in developing effective interpersonal relationships. Mutuality, respect for the patient, unconditional acceptance, and empathy are cornerstones of Sullivan's theory. The nurse who does not interact with the patient cannot demonstrate these cornerstones. Observations provide only objective data. Priority nursing diagnoses usually cannot be accurately established without subjective data from the patient. The other distracters relate to Maslow and behavioral theory. This item relates to an audience response question.

A nursing student expresses concerns that mental health nurses "lose all their clinical nursing skills." Select the best response by the mental health nurse a) "Psychiatric nurses practice in safer environments than other specialties. Nurse-to-patient ratios must be better because of the nature of the patients' problems." b) "Psychiatric nurses use complex communication skills as well as critical thinking to solve multidimensional problems. I am challenged by those situations." c) "That's a misconception. Psychiatric nurses frequently use high technology monitoring equipment and manage complex intravenous therapies." d) "Psychiatric nurses do not have to deal with as much pain and suffering as medical-surgical nurses do. That appeals to me."

B) "Psychiatric nurses use complex communication skills as well as critical thinking to solve multidimensional problems. I am challenged by those situations" The practice of psychiatric nursing requires a different set of skills than medical-surgical nursing, though there is substantial overlap. Psychiatric nurses must be able to help patients with medical as well as mental health problems, reflecting the holistic perspective these nurses must have. Nurse-patient ratios and workloads in psychiatric settings have increased, just like other specialties. Psychiatric nursing involves clinical practice, not just documentation. Psychosocial pain and suffering are as real as physical pain and suffering.

The parent of a child diagnosed with schizophrenia tearfully asks the nurse, "What could I have done differently to prevent this illness?" Select the nurse's best response. a. "Although schizophrenia results from impaired family relationships, try not to feel guilty. No one can predict how a child will respond to parental guidance." b. "Schizophrenia is a biological illness resulting from changes in how the brain and nervous system function. You are not to blame for your child's illness." c. "There is still hope. Changing your parenting style can help your child learn to cope effectively with the environment." d. "Most mental illnesses result from genetic inheritance. Your genes are more at fault than your parenting."

B) "Schizophrenia is a biological illness resulting from changes in how the brain and nervous system function. You are not to blame for your child's illness." The parent's comment suggests feelings of guilt or inadequacy. The nurse's response should address these feelings as well as provide information. Patients and families need reassurance that the major mental disorders are biological in origin and are not the "fault" of parents. One distracter places the burden of having faulty genes on the shoulders of the parents. The other distracters are neither wholly accurate nor reassuring.

A parent says, "My 2-year-old child refuses toilet training and shouts 'No!' when given directions. What do you think is wrong?" Select the nurse's best reply. a. "Your child needs firmer control. It is important to set limits now." b. "This is normal for your child's age. The child is striving for independence." c. "There may be developmental problems. Most children are toilet trained by age 2." d. "Some undesirable attitudes are developing. A child psychologist can help you develop a plan."

B) "This is normal for your child's age. The child is striving for independence." This behavior is conventional of a child around the age of 2 years, whose developmental task is to develop autonomy. The distracters indicate the child's behavior is abnormal.

A woman comes to the crisis intervention clinic and reports that her 16-year-old son uses drugs in the home and often assaults her. The nurse tells the client, A. "This is not an uncommon problem. Don't worry." B. "Together we will be able to work on this problem." C. "Now that you are asking for help, everything will be all right." D. "I have friends in law enforcement who can help us choose a solution."

B) "Together we will be able to work on this problem." The nurse takes an active collaborative role in problem resolution beginning with telling the client that a solution will be found.

Which statement made by the nurse demonstrates the best understanding of nonverbal communication? a. "The patient's verbal and nonverbal communication is often different." b. "When my patient responds to my question, I check for congruence between verbal and nonverbal communication to help validate the response." c. "If a patient is slumped in the chair, I can be sure he's angry or depressed." d. "It's easier to understand verbal communication that nonverbal communication."

B) "When my patient responds to my question, I check for congruence between verbal and nonverbal communication to help validate the response."

You have been working closely with a patient for the past month. Today he tells you he is looking forward to meeting with his new psychiatrist but frowns and avoids eye contact while reporting this to you. Which of the following responses would most likely be therapeutic? a. "A new psychiatrist is a chance to start fresh; I'm sure it will go well for you." b. "You say you look forward to the meeting, but you appear anxious or unhappy." c. "I notice that you frowned and avoided eye contact just now. Don't you feel well?" d. "I get the impression you don't really want to see your psychiatrist—can you tell me why?"

B) "You say you look forward to the meeting, but you appear anxious or unhappy."

A patient in the emergency department says, "Voices say someone is stalking me. They want to kill me because I developed a cure for cancer. I have a knife and will stab anyone who is a threat." Which aspects of the patient's mental health have the greatest and most immediate concern to the nurse? (Select all that apply.) a. Happiness b. Appraisal of reality c. Control over behavior d. Effectiveness in work e. Healthy self-concept

B) Appraisal of reality C) Control of behavior E) Healthy self-concept The aspects of mental health of greatest concern are the patient's appraisal of and control over behavior. The appraisal of reality is inaccurate. There are auditory hallucinations, delusions of persecution, and delusions of grandeur. In addition, the patient's control over behavior is tenuous, as evidenced by the plan to stab anyone who seems threatening. A healthy self-concept is lacking, as evidenced by the delusion of grandeur. Data are not present to suggest that the other aspects of mental health (happiness and effectiveness in work) are of immediate concern.

Which characteristic would be more applicable to a community mental health nurse than to a nurse working in an operating room? a. Kindness b. Autonomy c. Compassion d. Professionalism

B) Autonomy A community mental health nurse often works autonomously. Kindness, compassion, and professionalism apply to both nurses.

Which component of treatment of mental illness is specifically recognized by Quality and Safety Education for Nurses (QSEN)? a. All genomes are unique b. Care is centered on the patient. c. Healthy development is vital to mental health. d. Recovery occurs on a continuum from illness to health.

B) Care is centered on the patient The key areas of care promoted by QSEN are patient-centered care, teamwork and collaboration, evidence-based practice, quality improvement, safety, and informatics.

A nurse receives these three phone calls regarding a newly admitted patient.• The psychiatrist wants to complete an initial assessment.• An internist wants to perform a physical examination.• The patient's attorney wants an appointment with the patient.The nurse schedules the activities for the patient. Which role has the nurse fulfilled? a. Advocate b. Case manager c. Milieu manager d. Provider of care

B) Case Manager Nurses on psychiatric units routinely coordinate patient services, serving as case managers as described in this scenario. The role of advocate would require the nurse to speak out on the patient's behalf. The role of milieu manager refers to maintaining a therapeutic environment. Provider of care refers to giving direct care to the patient.

A patient says, "All my life I've been surrounded by stupidity. Everything I buy breaks because the entire American workforce is incompetent." This patient is experiencing a a. self-esteem deficit. b. cognitive distortion. c. deficit in motivation. d. deficit in love and belonging.

B) Cognitive distortion Automatic thoughts, or cognitive distortions, are irrational and lead to false assumptions and misinterpretations. See related audience response question.

A nurse assesses a confused older adult. The nurse experiences sadness and reflects, "This patient is like one of my grandparents ... so helpless." Which response is the nurse demonstrating? a. Transference b. Countertransference c. Catastrophic reaction d. Defensive coping reaction

B) Countertransference Countertransference is the nurse's transference or response to a patient that is based on the nurse's unconscious needs, conflicts, problems, or view of the world. See relationship to audience response question.

A nurse is caring for a client who reminds her of a negative person in her past. These memories cause the nurse to unconsciously displace negative feelings towards the client. The nurse should recognize that she is demonstrating which of the following behaviors? A) Suppression B) Countertransference C) Transference D) Inter-transference

B) Countertransference Countertransference: the nurse is treating the patient like someone they know Transference: The patient is treating the nurse like someone they know

A nurse wants to find a description of diagnostic criteria for anxiety disorders. Which resource would have the most complete information? a) Nursing Outcomes Classification (NOC) b) DSM-V c) The ANA's Psychiatric-Mental Health Nursing Scope and Standards of Practice d) ICD-10

B) DSM-V The DSM-V details the diagnostic criteria for psychiatric clinical conditions. It is the official guideline for diagnosing psychiatric disorders. The other references are good resources but do not define the diagnostic criteria.

The nurse is caring for a client who has a new diagnosis of colon cancer. Shortly after the client receives the diagnosis, the nurse enters the client's room and the client begins yelling, "I have received terrible care here and no one cares about me!" The nurse should recognize the client is demonstrating which of the following defense mechanisms? A) Denial B) Displacement C) Reaction formation D) Projection

B) Displacement redirection of thoughts, feelings and impulses from an object that causes anxiety to a safer, more acceptable one... patient is redirecting anxiety to staff that is providing care Reaction formation: behavior or emotion that is opposite of what client actually feels Projection: Client attributes undesired impulses to another person, place, or thing

Select the most critical question for the nurse to ask an adolescent who has threatened to takean overdose of pills. a. "Why do you want to kill yourself?" b. "Do you have access to medications?" c. "Have you been taking drugs and alcohol?" d. "Did something happen with your parents?"

B) Do you have access to medications?" The nurse must assess the patient's access to means to carry out the plan and, if there is access, alert the parents to remove from the home and take additional actions to assure the patient's safety. The information in the other questions may be important to ask but are not the most critical.

An adult outpatient diagnosed with major depressive disorder has a history of several suicide attempts by overdose. Given this patient's history and diagnosis, which antidepressant medication would the nurse expect to be prescribed? a. Amitriptyline b. Fluoxetine c. Desipramine d. Tranylcypromine sulfate

B) Fluoxetine Selective serotonin reuptake inhibitor antidepressants are very safe in overdosage situations, which is not true of the other medications listed. Given this patient's history of overdosing, it is important that the medication be as safe as possible in the event of another overdose of prescribed medication.

Carter, aged 36 years, comes to the crisis clinic for his first visit with complaints of not sleeping, anxiety, and excessive crying. He recently was fired suddenly from his job and 3 days later lost his home to a tornado that devastated the town he lives in. Which of the following statements regarding crisis accurately describes Carter's situation? A. He is experiencing low self-esteem from the job loss, as well as anger because of the loss of his home. B. He is experiencing both a situational and an adventitious crisis. C. He is experiencing ineffective coping and should be hospitalized for intensive therapy. D. He is experiencing a situational crisis with the added stress of financial burden.

B) He is experiencing both a situational and an adventitious crisis. It is possible to experience more than one type of crisis situation simultaneously, and as expected, the presence of more than one crisis further taxes individual coping skills. Carter lost his job (situational crisis) and also experienced the devastating effects of a tornado (adventitious crisis). The first option may be true but doesn't accurately describe the crisis criteria. There is nothing in the scenario suggesting he needs acute hospitalization at this time. He is experiencing not only a situational crisis, but an adventitious one as well, which makes coping more difficult.

Select the example of primary prevention. a. Assisting a person diagnosed with a serious mental illness to fill a pill-minder b. Helping school-age children identify and describe normal emotions c. Leading a psychoeducational group in a community care home d. Medicating an acutely ill patient who assaulted a staff person

B) Helping school-age children identify and describe normal emotions Primary preventions are directed at healthy populations with a goal of preventing health problems from occurring. Helping school-age children describe normal emotions people experience promotes coping, a skill that is needed throughout life. Assisting a person with serious and persistent mental illness to fill a pill-minder is an example of tertiary prevention. Medicating an acutely ill patient who assaulted a staff person is a secondary prevention. Leading a psychoeducational group in a community care home is an example of tertiary prevention.

A patient previously hospitalized for 2 weeks committed suicide the day after discharge.Which initial nursing measure will be most important regarding this event? a. Request the information technology manager to verify the patient's medical record is secure in the hospital information system. b. Hold a meeting for staff to provide support, express feelings, and identify overlooked clues or faulty judgments. c. Consult the hospital's legal department regarding potential consequences of the event. d. Document a report of a sentinel event in the patient's medical record.

B) Hold a meeting for staff to provide support, express feelings, and identify overlooked clues or faulty judgments. Support and an opportunity for staff to safely express feelings about the event should occur first. Interventions should help the staff come to terms with the loss and grow because of the incident. Identifying overlooked clues or faulty judgments will provide the groundwork for identifying changes needed in policies and procedures for future patients. Consulting the legal department is not an initial measure. A sentinel event report is not part of the medical record and can be prepared later. The other incorrect options will not control information or would result in unsafe care.

A nurse is caring for a client who has panic disorder and is experiencing anxiety at the panic level. Which of the following actions should the nurse take first? A) Identify the cause of the anxiety B) Instruct the client to take slow, deep breaths C) Teach the client how to use positive self-talk D) Explain the physical manifestations of anxiety to the client

B) Instruct the client to take slow, deep breaths

A novice nurse tells a mentor, "I want to convey to my patients that I am interested in them and that I want to listen to what they have to say." Which behaviors will be helpful in meeting the nurse's goal? (Select all that apply.) a. Sitting behind a desk, facing the patient b. Introducing self to a patient and identifying own role c. Maintaining control of discussions by asking direct questions d. Using facial expressions to convey interest and encouragement e. Assuming an open body posture and sometimes mirror imaging

B) Introducing self to a patient and identifying own role D) Using facial expressions to convey interest and encouragement E)Assuming an open body posture and sometimes mirror imaging Trust is fostered when the nurse gives an introduction and identifies his or her role. Facial expressions that convey interest and encouragement support the nurse's verbal statements to that effect and strengthen the message. An open body posture conveys openness to listening to what the patient has to say. Mirror imaging enhances patient comfort. A desk would place a physical barrier between the nurse and patient. A face-to-face stance should be avoided when possible and a less intense 90- or 120-degree angle used to permit either party to look away without discomfort.

Which activities represent the art of nursing? (Select all that apply.) a. Administering medications on time to a group of patients b. Listening to a new widow grieve her husband's death c. Helping a patient obtain groceries from a food bank d. Teaching a patient about a new medication e. Holding the hand of a frightened patient

B) Listening to a new widow grieve her husband's death C) Helping a patient obtain groceries from a food bank E) Holding the hand of a frightened patient Peplau described the science and art of professional nursing practice. The art component of nursing consists of the care, compassion, and advocacy nurses provide to enhance patient comfort and well-being. The science component of nursing involves the application of knowledge to understand a broad range of human problems and psychosocial phenomena, intervening to relieve patients' suffering and promote growth. See related audience response question.

A patient says, "I always feel good when I wear a size 2 petite." Which type of cognitive distortion is evident? a. Disqualifying the positive b. Overgeneralization c. Catastrophizing d. Personalization

B) Overgeneralization Automatic thoughts, or cognitive distortions, are irrational and lead to false assumptions and misinterpretations. The stem offers an example of overgeneralization. See related audience response question.

After a Category 5 tornado hits a community and destroys many homes and businesses, a community mental health nurse encourages victims to describe their memories and feelings about the event. This action by the nurse best demonstrates a. triage. b. primary prevention. c. psychosocial rehabilitation. d. psychiatric case management.

B) Primary prevention Tornado victims are at risk for psychiatric problems as a consequence of stress and trauma. Primary prevention occurs before any problem is manifested and seeks to reduce the incidence, or rate of new cases. Primary prevention may prevent or delay the onset of symptoms in predisposed individuals. Coping strategies and psychosocial support for vulnerable people are effective interventions in prevention. Disaster victims benefit from telling their story. Triage refers to the process of sorting out victims based on the immediacy of their needs for treatment. Psychosocial rehabilitation programs are designed to assist persons diagnosed with serious mental illness to develop living skills. Psychiatric case management refers to services to assist patients in finding housing or obtaining entitlements.

Which level of prevention activities would a nurse in an emergency department employ most often? a. Primary b. Secondary c. Tertiary

B) Secondary An emergency department nurse would generally see patients in crisis or with acute illness, so secondary prevention is used. Primary prevention involves preventing a health problem from developing, and tertiary prevention applies to rehabilitative activities.

Which change in the brain's biochemical function is most associated with suicidal behavior? a. Dopamine excess b. Serotonin deficiency c. Acetylcholine excess d. -aminobutyric acid deficiency

B) Serotonin deficiency Research suggests that low levels of serotonin may play a role in the decision to commit suicide. The other neurotransmitter alterations have not been implicated in suicidality.

Which student behavior is consistent with therapeutic communication? a. Offering your opinion when asked to convey support. b. Summarizing the essence of the patient's comments in your own words. c. Interrupting periods of silence before they become awkward for the patient. d. Telling the patient he did well when you approve of his statements or actions.

B) Summarizing the essence of the patient's comments in your own words.

A patient is fearful of riding on elevators. The therapist first rides an escalator with the patient. The therapist and patient then stand in an elevator with the door open for 5 minutes and later with the elevator door closed for 5 minutes. Which technique has the therapist used? a. Classic psychoanalytic therapy b. Systematic desensitization c. Rational emotive therapy d. Biofeedback

B) Systematic desensitization Systematic desensitization is a form of behavior modification therapy that involves the development of behavior tasks customized to the patient's specific fears. These tasks are presented to the patient while using learned relaxation techniques. The patient is incrementally exposed to the fear.

Which activity is appropriate for a nurse engaged exclusively in community-based primary prevention? a. Medication follow-up b. Teaching parenting skills c. Substance abuse counseling d. Making a referral for family therapy

B) Teaching parenting skills Primary prevention activities are directed to healthy populations to provide information for developing skills that promote mental health. The distracters represent secondary or tertiary prevention activities.

A nurse is assessing a client who is experiencing moderate-level anxiety. Which of the following findings should the nurse expect? A) The client has a heightened perceptual field B) The client has difficulty concentrating C) The client reports SOB D) The client reports a sense of impending doom

B) The client has difficulty concentrating A: mild anxiety C: panic level D: severe level

As a patient diagnosed with a mental illness is being discharged from a facility, a nurse invites the patient to the annual staff picnic. What is the best analysis of this scenario? a. The invitation facilitates dependency on the nurse. b. The nurse's action blurs the boundaries of the therapeutic relationship. c. The invitation is therapeutic for the patient's diversional activity deficit. d. The nurse's action assists the patient's integration into community living.

B) The nurse's action blurs the boundaries of the therapeutic relationship. The invitation creates a social relationship rather than a therapeutic relationship.

A patient says to the nurse, "My father has been dead for over 10 years, but talking to you is almost as comforting as the talks he and I had when I was a child." Which term applies to the patient's comment? a. Superego b. Transference c. Reality testing d. Counter-transference

B) Transference Transference refers to feelings a patient has toward the health care workers that were originally held toward significant others in his or her life. Counter-transference refers to unconscious feelings that the health care worker has toward the patient. The superego represents the moral component of personality; it seeks perfection.

A nurse influenced by Peplau's interpersonal theory works with an anxious, withdrawn patient. Interventions should focus on a. rewarding desired behaviors. b. use of assertive communication. c. changing the patient's self-concept. d. administering medications to relieve anxiety.

B) Use of assertive ommunication The nurse-patient relationship is structured to provide a model for adaptive interpersonal relationships that can be generalized to others. Helping the patient learn to use assertive communication will improve the patient's interpersonal relationships. The distracters apply to theories of cognitive, behavioral, and biological therapy.

Lilly, a single mother of four, comes to the crisis center 24 hours after an apartment fire in which all the family's household goods and clothing were lost. Lilly has no other family in the area. Her efforts to mobilize assistance have been disorganized, and she is still without shelter. She is distraught and confused. You assess the situation as: a. A maturational crisis. b. A situational crisis. c. An adventitious crisis. d. An existential crisis.

B) a situational crisis

The expected outcome at the conclusion of crisis intervention therapy is that the client will function A. at a higher level than before the crisis. B. at the precrisis level. C. only marginally below the precrisis level. D. without aid from identified support systems.

B) at the precrisis level The intent of crisis intervention is to return the individual to the precrisis level of functioning

A 12-year-old finds herself feeling anxious and overwhelmed and seeks out the school nurse to report that "Everything is changing . . . my body, the way the boys who were my friends are treating me, everything is so different." It is likely the child is A. describing personal identity disorder. B. experiencing a maturational crisis. C. potentially suicidal. D. mildly neurotic.

B) experiencing a maturational crisis. The maturational crisis of moving from childhood into adolescence may be difficult because many new coping skills are necessary.

A nurse inspects an inpatient psychiatric unit and finds that exits are free of obstructions, no one is smoking, and the janitor's closet is locked. These observations relate to a. coordinating care of patients. b. management of milieu safety. c. management of the interpersonal climate. d. use of therapeutic intervention strategies.

B) management of milieu safety. Nursing staff are responsible for all aspects of milieu management. The observations mentioned in this question directly relate to the safety of the unit. The other options, although part of the nurse's concerns, are unrelated to the observations cited.

Which assessment finding most clearly indicates that a patient may be experiencing a mental illness? The patient a) reports occasional sleeplessness and anxiety. b) reports a consistently sad, discouraged, and hopeless mood. c) is able to describe the difference between "as if" and "for real." d) perceives difficulty making a decision about whether to change jobs.

B) reports a consistently sad, discouraged, and hopeless mood The correct response describes a mood alteration, which reflects mental illness. The distracters describe behaviors that are mentally healthy or within the usual scope of human experience.

When a person becomes unemployed, he is likely to experience a(n) A. reactive crisis. B. situational crisis. C. adventitious crisis. D. substance abuse crisis.

B) situational crisis Situational crises arise from external sources. Examples are death of a loved one, divorce, marriage, or a change in health status.

A nurse introduces the matter of a contract during the first session with a new patient because contracts a. specify what the nurse will do for the patient. b. spell out the participation and responsibilities of each party. c. indicate the feeling tone established between the participants. d. are binding and prevent either party from prematurely ending the relationship.

B) spell out the participation and responsibilities of each party. A contract emphasizes that the nurse works with the patient rather than doing something for the patient. "Working with" is a process that suggests each party is expected to participate and share responsibility for outcomes. Contracts do not, however, stipulate roles or feeling tone, and premature termination is forbidden.

A nurse wants to enhance growth of a patient by showing positive regard. The nurse's action most likely to achieve this goal is a. making rounds daily. b. staying with a tearful patient. c. administering medication as prescribed. d. examining personal feelings about a patient.

B) staying with a tearful patient. Staying with a crying patient offers support and shows positive regard. Administering daily medication and making rounds are tasks that could be part of an assignment and do not necessarily reflect positive regard. Examining feelings regarding a patient addresses the nurse's ability to be therapeutic.

The nurse working with a client in crisis must initially assess for the client's A. self-report of feeling depressed. B. unrealistic report of a crisis-precipitating event. C. report of a high level of anxiety. D. admission that he or she is abusing drugs.

B) unrealistic report of a crisis-precipitating event. A person's equilibrium may be adversely affected by one or more of the following: an unrealistic perception of the precipitating event, inadequate situational supports, and inadequate coping mechanisms. These factors must be assessed when a crisis situation is evaluated because data gained from the assessment are used as guides for both the nurse and the client to set realistic and meaningful goals as well as to plan possible solutions to the problem situation.

A nurse wants to demonstrate genuineness with a patient diagnosed with schizophrenia. The nurse should a. restate what the patient says. b. use congruent communication strategies. c. use self-revelation in patient interactions. d. consistently interpret the patient's behaviors.

B) use congruent communication strategies. Genuineness is a desirable characteristic involving awareness of one's own feelings as they arise and the ability to communicate them when appropriate. The incorrect options are undesirable in a therapeutic relationship.

Which assessment finding for a patient diagnosed with serious and persistent mental illness and living in the community merits priority intervention by the psychiatric nurse? The patient a. receives social security disability income plus a small check from a trust fund every month. b. was absent from two of six planned Alcoholics Anonymous meetings in the past 2 weeks. c. lives in an apartment with two patients who attend partial hospitalization programs. d. has a sibling who was recently diagnosed with a mental illness.

B) was absent from two of six planned Alcoholics Anonymous meetings in the past 2 weeks. Patients who use alcohol or illegal substances often become medication non-adherent. Medication non-adherence, along with the disorganizing influence of substances on cellular brain function, promotes relapse. The distracters do not suggest problems.

An important question to ask during the assessment of a client diagnosed with anxiety disorder is: A. "How often do you hear voices?" B. "Have you ever considered suicide?" C. "How long has your memory been bad?" D. "Do your thoughts always seem jumbled?"

B. "Have you ever considered suicide?" The presence of anxiety may cause an individual to consider suicide as a means of finding comfort and peace. Suicide assessment is appropriate for any client with higher levels of anxiety.

Which assessment question would produce data that would help a nurse understand healing options acceptable to a client of a different culture? A. "Is there someone in your community who usually cures your illness?" B. "What usually helps people who have the same type of illness you have?" C. "What questions would you like to ask about your condition?" D. "What sorts of stress are you presently experiencing?"

B. "What usually helps people who have the same type of illness you have?" Asking about typical treatment seeks information about the "usual" cultural treatment of the disorder experienced by the client.

What can be said about the comorbidity of anxiety disorders? A. Anxiety disorders generally exist alone. B. A second anxiety disorder may coexist with the first. C. Anxiety disorders virtually never coexist with mood disorders. D. Substance abuse disorders rarely coexist with anxiety disorders.

B. A second anxiety disorder may coexist with the first. In many instances, when one anxiety disorder is present, a second one coexists. Clinicians and researchers have clearly shown that anxiety disorders frequently co-occur with other psychiatric problems. Major depression often co-occurs and produces a greater impairment with poorer response to treatment.

Blockage of dopamine transmission can lead to increased pituitary secretions of prolactin. In women, this hyperprolactinemia can result in: A. Dry mouth B. Amenorrhea C. Increased production of testosterone D. Blurred vision

B. Amenorrhea

Which drug group calls for nursing assessment for development of abnormal movement disorders among individuals who take therapeutic dosages? A. SSRIs B. Antipsychotics C. Benzodiazepines D. Trycyclic antidepressants

B. Antipsychotics

Which ethical principle refers to the individual's right to make his or her own decisions? A. Beneficence B. Autonomy C. Veracity D. Fidelity

B. Autonomy Rationale: Autonomy refers to self-determination, or the right to make one's own decisions.

What ethical principle is supported when a nurse witnesses the informed consent for electroconvulsive therapy from a depressed client? A. Beneficence B. Autonomy C. Justice D. Fidelity

B. Autonomy Rationale: Autonomy refers to self-determination. One way to exercise self-determination is to make decisions about one's care.

A client who presents no danger to himself or to others is forced to take medication against his will. This situation represents: A. Assault. B. Battery. C. Defamation. D. Invasion of privacy.

B. Battery. Rationale: Battery is the harmful, nonconsensual touching of another person. Forceful administration of medication constitutes battery.

Studies of clients diagnosed with posttraumatic stress disorder suggest that the stress response of which of the following is considered abnormal? A. Brainstem B. Hypothalamus-pituitary-adrenal system C. Frontal lobe D. Limbic system

B. Hypothalamus-pituitary-adrenal system Studies of clients with posttraumatic stress disorder suggest that the stress response of the hypothalamus-pituitary-adrenal system is abnormal.

Sophie, aged 27 years, has a diagnosis of paranoid schizophrenia. She stopped taking her medications and believes that she is to be taken by the aliens to live with them on another planet. She was observed walking through traffic on a busy road, and then was found climbing the railing on a bridge, to "be ready for them to take me in their ship." Sophie is hospitalized. During your shift she begins running up and down the halls, banging her head on the walls, and yelling, "Get them out of my head!" On what basis can Sophie be medicated against her will? A. If Sophie has taken the medication in the past and has had no adverse effects B. If Sophie may cause imminent harm to herself or others C. If Sophie still has the capacity to make an informed decision regarding medication D. If Sophie is provided education regarding the medication before administration of the medication

B. If Sophie may cause imminent harm to herself or others Rationale: A patient may be medicated against his or her will without a court hearing in an emergency if the patient poses a danger to himself or herself or to others. The other options are not legally valid reasons to give medication against a patient's will.

If a client is placed in seclusion and held there for 24 hours without a written order or examination by a physician, the client has experienced: A. Battery. B. Defamation of character. C. False imprisonment. D. Assault.

C. False imprisonment. Rationale: False imprisonment is the arbitrary holding of a client against his or her will. When seclusion is ordered, it is not invoked arbitrarily, but after other less restrictive measures have failed. If the client is secluded without the medical order, the measure cannot be proven as instituted for medically sound reasons.

Jerry is a 72-year-old patient with Parkinson's disease and anxiety. He is living by himself and has had several falls lately. His provider orders lorazepam, 1 mg PO bid, for anxiety. You question this order because: A. Jerry may become addicted faster than younger patients. B. Jerry is at risk for falls. C. Jerry has a history of nonadherence with medications. D. Jerry should be treated with cognitive therapies rather than medication because of his advanced age.

B. Jerry is at risk for falls. An important nursing intervention is to monitor for side effects of the benzodiazepines, including sedation, ataxia, and decreased cognitive function. In a patient who has a history of falls, lorazepam would be contraindicated because it may cause sedation and ataxia leading to more falls. There is no evidence to suggest that elderly patients become addicted faster than younger patients. A history of nonadherence would not lead to you to question this drug order. Medication and other therapies are used congruently with all age levels.

You are working on an inpatient psychiatric unit and caring for Elizabeth, who is becoming agitated. You speak with Elizabeth one to one in a private setting, find out the reason for the agitation, and then assist Elizabeth with ways to calm down, possibly including prn medication to prevent further escalation of Elizabeth's agitation, which could lead to seclusion and/or restraints. You are making care decisions based on: A. Writ of habeas corpus .B. Least restrictive alternative doctrine. C. Veracity. D. Bioethics.

B. Least restrictive alternative doctrine. Rationale: Least restrictive alternative doctrine is described as using the least drastic means of achieving a specific goal. By doing the actions described you are possibly preventing the more restrictive setting of seclusion and/or restraints. Writ of habeas corpus is a legal term meaning a written order "to free the person." Veracity is one of the five ethical principles or guidelines. Bioethics refers to ethics in a health care setting.

Which therapeutic intervention can the nurse implement personally to help a client diagnosed with a mild anxiety disorder regain control? A. Flooding B. Modeling C. Thought stopping D. Systematic desensitization

B. Modeling Modeling calm behavior in the face of anxiety or unafraid behavior in the presence of a feared stimulus are interventions that can be independently used. The other options require agreement of the treatment team.

Ms. Wong, aged 52 years, comes to the emergency room with severe anxiety. She was raised in China but immigrated to the United States at age 40 years. She was recently fired from her job because of a major error in the accounting department that she managed. Ms. Wong's aged parents live with her. Ms. Wong states, "I am a failure." Which of the following statements may accurately assess the basis for Ms. Wong's anxiety and feelings of failure? A. Ms. Wong may feel that she has let herself down since she did not achieve her personal goals in the workplace. B. Ms. Wong may feel that she has shamed the family by being fired and may no longer be able to provide for them. C. Ms. Wong may feel personally inadequate since she failed in her quest for independence and self-reliance. D. Ms. Wong may be feeling anxiety because in her family's traditions her failure may result in a changed fate.

B. Ms. Wong may feel that she has shamed the family by being fired and may no longer be able to provide for them. Eastern tradition, such as in China, where Ms. Wong is from, sees the family as the basis for one's identity, and family interdependence as the norm. The views expressed in options a and c demonstrate Western tradition where self-reliance, individuality, and autonomy are highly valued. In the Eastern view one is born into an unchangeable fate.

Data concerning client age, sex, education, and income should be the focus of an assessment in order to best understand cultural issues related to A. health practices. B. power and control. C. psychological stability. D. assimilation and conformity.

B. Power and control Power and control are often products of culturally determined beliefs about who should hold power. In many cultures the elderly are venerated. In other cultures women are virtually powerless. For some cultures, higher education equates with power.

If a nurse is charged with leaving a suicidal client unattended, it is being suggested that the nurse's behavior has violated the ethical principle of A. Autonomy. B. Veracity. C. Fidelity. D. Justice.

C. Fidelity. Rationale: Fidelity refers to being "true" or faithful to one's obligations to the client. Client abandonment would be a violation of fidelity.

Josefina Juarez, aged 36 years, comes to the mental health clinic where you work after being referred by her primary care provider. Josefina came to live in the United States from Brazil 5 years ago. She is now a single mother to 6 children, ages 2 to 15, following the death of her husband last year. During the initial intake assessment, Josefina tells you her problem is that she has headaches and backaches "almost every day" and "can't sleep at night." She shakes her head no and looks away when asked about anxiety or depression and states she does not know why she was referred to the mental health clinic. You recognize that Josefina may be exhibiting: A. regression. B. somatization. C. enculturation. D. assimilation.

B. Somatization Somatization is described as experiencing and expressing emotional or psychological distress as physical symptoms. Regression is a defense mechanism meaning to begin to function at a lower or previous level of functioning. Enculturation refers to how cultural beliefs, practices, and norms are communicated to its members. Assimilation refers to a situation in which immigrants adapt to and absorb the practices and beliefs of a new culture until these customs are more natural than the ones they learned in their homeland.

An obsession is defined as: A. thinking of an action and immediately taking the action. B. a recurrent, persistent thought or impulse. C. an intense irrational fear of an object or situation. D. a recurrent behavior performed in the same manner.

B. a recurrent, persistent thought or impulse. Obsessions are thoughts, impulses, or images that persist and recur so that they cannot be dismissed from the mind.

The nurse anticipates that the nursing history of a client diagnosed with obsessive compulsive disorder (OCD) will reveal: A. a history of childhood trauma. B. a sibling with the disorder. C. an eating disorder. D. a phobia as well.

B. a sibling with the disorder. Research shows that first-degree biological relatives of those with OCD have a higher frequency of the disorder than exists in the general population.

Stella brings her mother, Dorothy, to the mental health outpatient clinic. Dorothy has a history of anxiety. Stella and Dorothy both give information for the assessment interview. Stella states, "My mother lives with me since my dad died 6 months ago. For the past couple of months, every time I need to leave the house for work or anything else, Mom becomes extremely anxious and cries that something terrible is going to happen to me. She seems OK except for these times, but it's affecting my ability to go to work." You suspect: A. panic disorder. B. adult separation anxiety disorder. C. agoraphobia. D. social anxiety disorder.

B. adult separation anxiety disorder. People with separation anxiety disorder exhibit developmentally inappropriate levels of concern over being away from a significant other. There may also be fear that something horrible will happen to the other person. Adult separation anxiety disorder may begin in childhood or adulthood. The scenario doesn't describe panic disorder. Agoraphobia is characterized by intense, excessive anxiety or fear about being in places or situations from which escape might be difficult or embarrassing or in which help might not be available. Social anxiety disorder, also called social phobia, is characterized by severe anxiety or fear provoked by exposure to a social or a performance situation that will be evaluated negatively by others.

The defense mechanisms that can only be used in healthy ways include: A. suppression and humor. B. altruism and sublimation. C. idealization and splitting. D. reaction formation and denial.

B. altruism and sublimation. Altruism and sublimation are known as mature defenses. They cannot be used in unhealthy ways. Altruism results in resolving emotional conflicts by meeting the needs of others, and sublimation substitutes socially acceptable activity for unacceptable impulses.

The initial nursing action for a newly admitted anxious client is to: A. assess the client's use of defense mechanisms. B. assess the client's level of anxiety. C. limit environmental stimuli. D. provide antianxiety medication.

B. assess the client's level of anxiety. The priority nursing action is the assessment of the client's anxiety level.

A nurse interacts with an outpatient who has a history of multiple suicide attempts. Select the most helpful response for a nurse to make when the patient states, "I am considering committing suicide." a. "I'm glad you shared this. Please do not worry. We will handle it together." b. "I think you should admit yourself to the hospital to keep you safe." c. "Bringing up these feelings is a very positive action on your part." d. "We need to talk about the good things you have to live for."

C) "Bringing up these feelings is a very positive action on your part." The correct response gives the patient reinforcement, recognition, and validation for making a positive response rather than acting out the suicidal impulse. It gives neither advice nor false reassurance, and it does not imply stereotypes such as "You have a lot to live for." It uses the patient's ambivalence and sets the stage for more realistic problem solving.

A nurse and patient construct a no-suicide contract. Select the preferable wording. a. "I will not try to harm myself during the next 24 hours." b. "I will not make a suicide attempt while I am hospitalized." c. "For the next 24 hours, I will not in any way attempt to harm or kill myself." d. "I will not kill myself until I call my primary nurse or a member of the staff."

C) "For the next 24 hours, I will not in any way attempt to harm or kill myself." The correct answer leaves no loopholes. The wording about not harming oneself and not making an attempt leaves loopholes or can be ignored by the patient who thinks "I am not going to harm myself, I am going to kill myself" or "I am not going to attempt suicide, I am going to commit suicide." A patient may call a therapist and leave the telephone to carry out the suicidal plan.

A nurse assesses a patient who reports a 3-week history of depression and periods of uncontrolled crying. The patient says, "My business is bankrupt, and I was served with divorce papers." Which subsequent statement by the patient alerts the nurse to a concealed suicidal message? a. "I wish I were dead." b. "Life is not worth living." c. "I have a plan that will fix everything." d. "My family will be better off without me."

C) "I have a plan that will fix everything." Verbal clues to suicide may be overt or covert. The incorrect options are overt references to suicide. The correct option is more veiled. It alludes to the patient's suicide as being a way to"fix everything" but does not say it outright.

A nurse enters a client's room and observes that the client is agitated and pacing rapidly. The client looks at the nurse and says, "Back off. Leave me alone!" Which of the following statements should the nurse make? A) "I demand that you calm down now. Your behavior is unacceptable" B) "I will close the door to provide privacy, and you can tell me what is bothering you" C) "I will give you space if you calm down. Tell me what is causing you to feel so tense" D) I will leave you alone for a few minutes while you try to control yourself"

C) "I will give you space if you calm down. Tell me what is causing you to feel so tense" "Tell me" = therapeutic Never close the door or leave the patient alone

A family has a long history of conflicted relationships among the members. Which family member's comment best reflects a mentally healthy perspective? a) "I've made mistakes but everyone else in this family has also." b) "I remember joy and mutual respect from our early years together." c) "I will make some changes in my behavior for the good of the family." d) "It's best for me to move away from my family. Things will never change."

C) "I will make some changes in my behavior for the good of the family" The correct response demonstrates the best evidence of a healthy recognition of the importance of relationships. Mental health includes rational thinking, communication skills, learning, emotional growth, resilience, and self-esteem. Recalling joy from earlier in life may be healthy, but the correct response shows a higher level of mental health. The other incorrect responses show blaming and avoidance.

As a nurse escorts a patient being discharged after treatment for major depression, the patient gives the nurse a necklace with a heart pendant and says, "Thank you for helping mend my broken heart." Which is the nurse's best response? a. "Accepting gifts violates the policies and procedures of the facility." b. "I'm glad you feel so much better now. Thank you for the beautiful necklace." c. "I'm glad I could help you, but I can't accept the gift. My reward is seeing you with a renewed sense of hope." d. "Helping people is what nursing is all about. It's rewarding to me when patients recognize how hard we work."

C) "I'm glad I could help you, but I can't accept the gift. My reward is seeing you with a renewed sense of hope." Accepting a gift creates a social rather than therapeutic relationship with the patient and blurs the boundaries of the relationship. A caring nurse will acknowledge the patient's gesture of appreciation, but the gift should not be accepted. See relationship to audience response question.

A nurse presents a community education program about mental illness. Which comment by a participant best demonstrates a correct understanding of mental illness from a biological perspective? a. "Some people experience life events so traumatic that they cannot be overcome." b. "Disturbed and conflicted family relationships are usually a starting place for mental illness." c. "My friend has had bipolar disorder for years and many problems have resulted. It's not her fault." d. "Mental illness is the result of developmental complications that cause a person not to grow to their full potential."

C) "My friend has had bipolar disorder for years and many problems have resulted. It's not her fault." The correct response demonstrates an understanding that mental illness is physical in origin. The physical origins of mental illness are aspects of the biological model. The incorrect responses assign the origins of mental illness to interpersonal relationships and traumatic events.

Which statements most clearly reflect the stigma of mental illness? (Select all that apply.) a. "Many mental illnesses are hereditary." b. "Mental illness can be evidence of a brain disorder." c. "People claim mental illness so they can get disability checks." d. "Mental illness results from the breakdown of American families." e. "If people with mental illness went to church, their symptoms would disappear."

C) "People claim mental illness so they can get disability checks." D) "Mental illness results from the breakdown of American families." E) "If people with mental illness went to church, their symptoms would disappear." Stigma is represented by judgmental remarks that discount the reality and validity of mental illness. Many mental illnesses are genetically transmitted. Neuroimaging can show changes associated with some mental illnesses.

The spouse of a patient diagnosed with schizophrenia says, "I don't understand how events from childhood have anything to do with this disabling illness." Which response by the nurse will best help the spouse understand the cause of this disorder? a. "Psychological stress is the basis of most mental disorders." b. "This illness results from developmental factors rather than stress." c. "Research shows that this condition more likely has a biological basis." d. "It must be frustrating for you that your spouse is sick so much of the time."

C) "Research shows that this condition more likely has a biological basis" Many of the most prevalent and disabling mental disorders have strong biological influences. Genetics are only one part of biological factors. Empathy does not address increasing the spouse's level of knowledge about the cause of the disorder. The other distracters are not established facts.

A patient says, "I'm still on restriction, but I want to attend some off-unit activities. Would you ask the doctor to change my privileges?" What is the nurse's best response? a. "Why are you asking me when you're able to speak for yourself?" b. "I will be glad to address it when I see your doctor later today." c. "That's a good topic for you to discuss with your doctor." d. "Do you think you can't speak to a doctor?"

C) "That's a good topic for you to discuss with your doctor." Nurses should encourage patients to work at their optimal level of functioning, which in turn promotes autonomy. A nurse does not act for the patient unless it is necessary. Acting for a patient increases feelings of helplessness and dependency.

An adult dies in a tragic accident. Afterward, the siblings plan a funeral service. Which statement by a sibling best indicates a sense of self-actualization? a. "Of all of us, I am the most experienced with planning these types of events." b. "Funerals are supposed to be conducted quietly, respectfully, and according to a social protocol." c. "This death was unfair but I hope we can plan a service that everyone feels is a celebration of life." d. "This death was probably the consequence of years of selfish and inconsiderate behavior by our sibling."

C) "This death was unfair but I hope we can plan a service that everyone feels is a celebration of life." The correct response shows an accurate perception of reality as well as a focus on solving the problem in a way that involves others. These factors are characteristic of self-actualization. The incorrect responses demonstrate self-centeredness, rigidity, and blaming which are characteristic of a failure to achieve self-actualization.

A nurse is counseling a client who seems relaxed initially, but then becomes restless and begins wringing his hands. The nurse states that the client seems tense, and the client agrees. Which of the following statements should the nurse make? A) "Did i say something wrong that made you feel tense?" B) "Do you often feel tense when you are talking to a healthcare provider?" C) "What were we discussing when you began to feel uncomfortable?" D) "It is okay to feel nervous during our counseling sessions"

C) "What were we discussing when you began to feel uncomfortable" Don't ask yes or no questions; unless it is regarding self harm

James is a 42-year-old patient with schizophrenia. He approaches you as you arrive for day shift and anxiously reports, "Last night, demons came to my room and tried to rape me." Which response would be most therapeutic? a. "There are no such things as demons. What you saw were hallucinations." b. "It is not possible for anyone to enter your room at night. You are safe here." c. "You seem very upset. Please tell me more about what you experienced last night." d. "That must have been very frightening, but we'll check on you at night and you'll be safe."

C) "You seem very upset. Please tell me more about what you experienced last night."

A nurse is caring for a client who has depression and started taking paroxetine one week ago. The client states to the nurse, "My family would be better off without me." Which of the following responses should the nurse make? A) "Why do you feel your family would be better off without you?" B) "Many people feel this way when they are depressed" C) "You sound upset. Are you thinking of hurting yourself?" D) "Your medication hasn't started working yet. When it does, you'll be feeling differently

C) "You sound upset. Are you thinking of hurting yourself?"

A crisis is so acutely uncomfortable to the individual that it is likely to self-resolve in A. 1 to 10 days. B. 1 to 3 weeks. C. 4 to 6 weeks. D. 3 to 4 months

C) 4 to 6 weeks At 4 to 6 weeks, the individual is making accommodations and adjustments to relieve anxiety, and the crisis is no longer a crisis.

When a new bill introduced in Congress reduces funding for care of persons diagnosed with mental illness, a group of nurses write letters to their elected representatives in opposition to the legislation. Which role have the nurses fulfilled? a) Recovery b) Attending c) Advocacy d) Evidence-based practice

C) Advocacy An advocate defends or asserts another's cause, particularly when the other person lacks the ability to do that for self. Examples of individual advocacy include helping patients understand their rights or make decisions. On a community scale, advocacy includes political activity, public speaking, and publication in the interest of improving the human condition. Since funding is necessary to deliver quality programming for persons with mental illness, the letter-writing campaign advocates for that cause on behalf of patients who are unable to articulate their own needs.

A patient was hospitalized for 24 hours after a reaction to a psychotropic medication. While planning discharge, the case manager learned that the patient received a notice of eviction immediately prior to admission. Select the case manager's most appropriate action. a. Postpone the patient's discharge from the hospital. b. Contact the landlord who evicted the patient to further discuss the situation. c. Arrange a temporary place for the patient to stay until new housing can be arranged. d. Determine whether the adverse medication reaction was genuine because the patient had nowhere to live.

C) Arrange a temporary place for the patient to stay until new housing can be arranged. The case manager should intervene by arranging temporary shelter for the patient until an apartment can be found. This activity is part of the coordination and delivery of services that falls under the case manager role. None of the other options is a viable alternative.

Which nursing diagnosis would be most useful for clients with anxiety disorders? A. Excess fluid volume B. Disturbed body image C. Ineffective role performance D. Disturbed personal identity

C. Ineffective role performance Anxiety disorders often interfere with the usual role performance of clients. Consider the client with agoraphobia who cannot go to work, or the client with obsessive-compulsive disorder who devotes time to the ritual rather than to parenting.

A suspicious, socially isolated patient lives alone, eats one meal a day at a local shelter, and spends the remaining daily food allowance on cigarettes. Select a community psychiatric nurse's best initial action. a. Explore ways to help the patient stop smoking. b. Report the situation to the manager of the shelter. c. Assess the patient's weight; determine foods and amounts eaten. d. Arrange hospitalization for the patient in order to formulate a new treatment plan.

C) Assess the patient's weight; determine foods and amounts eaten. Assessment of biopsychosocial needs and general ability to live in the community is called for before any other action is taken. Both nutritional status and income adequacy are critical assessment parameters. A patient may be able to maintain adequate nutrition while eating only one meal a day. The rule is to assess before taking action. Hospitalization may not be necessary. Smoking cessation strategies can be pursued later.

Which intervention will the nurse recommend for the distressed family and friends of someone who has committed suicide? a. Participating in reminiscence therapy b. Psychological postmortem assessment c. Attending a self-help group for survivors d. Contracting for at least two sessions of group therapy

C) Attending a self-help group for survivors Survivors need outlets for their feelings about the loss and the deceased person. Self-help groups provide peer support while survivors work through feelings of loss, anger, and guilt.Psychological postmortem assessment would not provide the support necessary to work through feelings of loss associated with the suicide. Reminiscence therapy is not geared to loss resolution. Contracting for two sessions of group therapy would not provide sufficient time to work through the issues associated with a death by suicide.

An adult says, "I never know the answers," and "My opinion does not count." Which psychosocial crisis was unsuccessfully resolved for this adult?' a. Initiative versus guilt b. Trust versus mistrust c. Autonomy versus shame and doubt d. Generativity versus self-absorption

C) Autonomy versus shame and doubt These statements show severe self-doubt, indicating that the crisis of gaining control over the environment was not met successfully. Unsuccessful resolution of the crisis of initiative versus guilt results in feelings of guilt. Unsuccessful resolution of the crisis of trust versus mistrust results in poor interpersonal relationships and suspicion of others. Unsuccessful resolution of the crisis of generativity versus self-absorption results in self-absorption that limits the ability to grow as a person.

When assessing a patient's plan for suicide, what aspect has priority? a. Patient's financial and educational status b. Patient's insight into suicidal motivation c. Availability of means and lethality of method d. Quality and availability of patient's social support

C) Availability of means and lethality of method If a person has plans that include choosing a method of suicide readily available and if the method is one that is lethal (i.e., will cause the person to die with little probability for intervention), the suicide risk is high. These areas provide a better indication of risk than the areas mentioned in the other options. See relationship to audience response question.

A psychotherapist works with an anxious, dependent patient. Which strategy is most consistent with psychoanalytic psychotherapy? a. Identifying the patient's strengths and assets b. Praising the patient for describing feelings of isolation c. Focusing on feelings developed by the patient toward the therapist d. Providing psychoeducation and emphasizing medication adherence

C) Focusing on feelings developed by the patient toward the therapist Positive or negative feelings of the patient toward the therapist indicate transference. Transference is a psychoanalytic concept that can be used to explore previously unresolved conflicts. The distracters relate to biological therapy and supportive psychotherapy. Use of psychoeducational materials is a common "homework" assignment used in cognitive therapy.

A college student who failed two tests cried for hours and then tried to telephone a parent but got no answer. The student then gave several expensive sweaters to a roommate and asked to be left alone for a few hours. Which behavior provides the strongest clue of an impending suicide attempt? a. Calling parents b. Excessive crying c. Giving away sweaters d. Staying alone in dorm room

C) Giving away sweaters Giving away prized possessions may signal that the individual thinks he or she will have no further need for the item, such as when a suicide plan has been formulated. Calling parents, remaining in a dorm, and crying do not provide direct clues to suicide.

A category 5 tornado occurred in a community of 400 people. Many homes and businesses were destroyed. In the 2 years following the disaster, 140 individuals were diagnosed with posttraumatic stress disorder (PTSD). Which term best applies to these newly diagnosed cases? a. Prevalence b. Comorbidity c. Incidence d. Parity

C) Incidence Incidence refers to the number of new cases of mental disorders in a healthy population within a given period of time. Prevalence describes the total number of cases, new and existing, in a given population during a specific period of time, regardless of when they became ill. Parity refers to equivalence, and legislation required insurers that provide mental health coverage to offer annual and lifetime benefits at the same level provided for medical-surgical coverage. Comorbidity refers to having more than one mental disorder at a time.

The case manager plans to discuss the treatment plan with a patient's family. Select the case manager's first action. a. Determine an appropriate location for the conference. b. Support the discussion with examples of the patient's behavior. c. Obtain the patient's permission for the exchange of information. d. Determine which family members should participate in the conference.

C) Obtain the patient's permission for the exchange of information The case manager must respect the patient's right to privacy, which extends to discussions with family. Talking to family members is part of the case manager's role. Actions identified in the distracters occur after the patient has given permission.

A patient usually watches television all day, seldom going out in the community or socializing with others. The patient says, "I don't know what to do with my free time." Which member of the treatment team would be most helpful to this patient? a. Psychologist b. Social worker c. Recreational therapist d. Occupational therapist

C) Recreational Therapist Recreational therapists help patients use leisure time to benefit their mental health. Occupational therapists assist with a broad range of skills, including those for employment. Psychologists conduct testing and provide other patient services. Social workers focus on the patient's support system.

The unit secretary receives a phone call from the health insurer for a hospitalized patient. The caller seeks information about the patient's projected length of stay. How should the nurse instruct the unit secretary to handle the request? a. Obtain the information from the patient's medical record and relay it to the caller. b. Inform the caller that all information about patients is confidential. c. Refer the request for information to the patient's case manager. d. Refer the request to the health care provider.

C) Refer the request for information to the patient's case manager. The case manager usually confers with insurers and provides the treatment team with information about available resources. The unit secretary should be mindful of patient confidentiality and should neither confirm that the patient is an inpatient nor disclose other information.

Which issues should a nurse address during the first interview with a patient with a psychiatric disorder? a. Trust, congruence, attitudes, and boundaries b. Goals, resistance, unconscious motivations, and diversion c. Relationship parameters, the contract, confidentiality, and termination d. Transference, countertransference, intimacy, and developing resources

C) Relationship parameters, the contract, confidentiality, and termination Relationship parameters, the contract, confidentiality, and termination are issues that should be considered during the orientation phase of the relationship. The remaining options are issues that are dealt with later.

The parent of a 4-year-old rewards and praises the child for helping a sibling, being polite, and using good manners. These qualities are likely to be internalized and become part of which system of the personality? a. Id b. Ego c. Superego d. Preconscious

C) Superego The superego contains the "shoulds," or moral standards internalized from interactions with significant others. Praise fosters internalization of desirable behaviors. The id is the center of basic instinctual drives, and the ego is the mediator. The ego is the problem-solving and reality-testing portion of the personality that negotiates solutions with the outside world. The preconscious is a level of awareness from which material can be retrieved easily with conscious effort. This item relates to an audience response question.

A college student received an invitation to attend the wedding of a close friend who lives across the country. The student is afraid of flying. Which type of therapy would be most helpful for this patient? a. Psychoanalysis b. Aversion therapy c. Systematic desensitization d. Short-term dynamic therapy

C) Systemic desensitization Systematic desensitization is a type of therapy aimed at extinguishing a specific behavior, such as the fear of flying. Psychoanalysis and short-term dynamic therapy seek to uncover conflicts. Aversion therapy involves use of a noxious stimulus, punishment, and avoidance.

After several therapeutic encounters with a patient who recently attempted suicide, which occurrence should cause the nurse to consider the possibility of countertransference? a. The patient's reactions toward the nurse seem realistic and appropriate. b. The patient states, "Talking to you feels like talking to my parents." c. The nurse feels unusually happy when the patient's mood begins to lift. d. The nurse develops a trusting relationship with the patient.

C) The nurse feels unusually happy when the patient's mood begins to lift. Strong positive or negative reactions toward a patient or over-identification with the patient indicate possible countertransference. Nurses must carefully monitor their own feelings and reactions to detect countertransference and then seek supervision. Realistic and appropriate reactions from a patient toward a nurse are desirable. One incorrect response suggests transference. A trusting relationship with the patient is desirable. See relationship to audience response question.

Tori is the nurse working with Carter and other members of the community after the tornado. As the weeks go by, she begins to feel anxious and distressed. She speaks to her nurse mentor about her feelings. Which of the following may Tori be experiencing? A. Reactionary grief B. Maturational crisis C. Vicarious traumatization D. Transference

C) Vicarious traumatization Even experienced nurses working in disaster situations can become overwhelmed when witnessing catastrophes such as loss of human life or mass destruction of people's homes and belongings (e.g., floods, fires, tornadoes). Researchers have found that mental health care providers may experience psychological distress from working with traumatized populations, a phenomenon of secondary traumatic stress or "vicarious traumatization." Reactionary grief does not describe secondary stress from working with such populations. A maturational crisis arises from disruption of a developmental stage. Transference describes feelings displaced onto the nurse or therapist by the patient.

During which phase of the nurse-patient relationship can the nurse anticipate that identified patient issues will be explored and resolved? a. Preorientation b. Orientation c. Working d. Termination

C) Working During the working phase, the nurse strives to assist the patient in making connections among dysfunctional behaviors, thinking, and emotions and offers support while alternative coping behaviors are tried.

When a tornado results in the loss of homes, businesses, and life, the town residents are likely to experience a(n) A. maturational crisis. B. situational crisis. C. adventitious crisis. D. endogenous crisis.

C) adventitious crisis An adventitious crisis is unplanned, accidental, and not part of everyday life. Examples are disasters and crimes of violence.

Which behavior shows that a nurse values autonomy? The nurse a. suggests one-on-one supervision for a patient who has suicidal thoughts. b. informs a patient that the spouse will not be in during visiting hours. c. discusses options and helps the patient weigh the consequences. d. sets limits on a patient's romantic overtures toward the nurse.

C) discusses options and helps the patient weigh the consequences. A high level of valuing is acting on one's belief. Autonomy is supported when the nurse helps a patient weigh alternatives and their consequences before the patient makes a decision. Autonomy or self-determination is not the issue in any of the other behaviors.

When the nurse reads the medical record and learns that a client has agreed to receive treatment and abide by hospital rules, the correct assumption is that the client was admitted: A. Per legal requirements. B. For a non-emergency. C. Voluntarily. D. Involuntarily.

C. Voluntarily. Rationale: Voluntary admission occurs when the client is willing to be admitted and agrees to comply with hospital and unit rules.

Termination of a therapeutic nurse-patient relationship has been successful when the nurse a. avoids upsetting the patient by shifting focus to other patients before the discharge. b. gives the patient a personal telephone number and permission to call after discharge. c. discusses with the patient changes that happened during the relationship and evaluates outcomes. d. offers to meet the patient for coffee and conversation three times a week after discharge.

C) discusses with the patient changes that happened during the relationship and evaluates outcomes. Summarizing and evaluating progress help validate the experience for the patient and the nurse and facilitate closure. Termination must be discussed; avoiding discussion by spending little time with the patient promotes feelings of abandonment. Successful termination requires that the relationship be brought to closure without the possibility of dependency-producing ongoing contact

A tearful, anxious patient at the outpatient clinic reports, "I should be dead." The initial task of the nurse conducting the assessment interview is to a. assess lethality of suicide plan. b. encourage expression of anger. c. establish trust with the patient. d. determine risk factors for suicide.

C) establish trust with the patient. This scenario presents a potential crisis. Establishing trust facilitates a therapeutic alliance that will allow the nurse to obtain relevant assessment data such as the presence of a suicide plan, lethality of plan, and presence of risk factors for suicide.

A patient diagnosed with schizophrenia had an exacerbation related to medication non-adherence and was hospitalized for 5 days. The patient's thoughts are now more organized and discharge is planned. The patient's family says, "It's too soon for discharge. We will just go through all this again." The nurse should a. ask the case manager to arrange a transfer to a long-term care facility. b. notify hospital security to handle the disturbance and escort the family off the unit. c. explain that the patient will continue to improve if the medication is taken regularly. d. contact the health care provider to meet with the family and explain the discharge rationale.

C) explain that the patient will continue to improve if the medication is taken regularly. Patients do not stay in a hospital until every symptom disappears. The nurse must assume responsibility to advocate for the patient's right to the least restrictive setting as soon as the symptoms are under control and for the right of citizens to control health care costs. The health care provider will use the same rationale. Shifting blame will not change the discharge. Security is unnecessary. The nurse can handle this matter.

The nurse caring for a client in crisis shows signs of a problematic nurse-client relationship by A. offering to change the time of the counseling session for the second time in 3 weeks. B. experiencing frustration about the decisions the client is making. C. giving the client permission to call him or her at home when the client "needs to talk." D. suggesting that the client attend an extra counseling session each month.

C) giving the client permission to call him or her at home when the client "needs to talk." The behavior in option C is a reaction to the nurse's need to be needed and undermines the client's sense of self-reliance.

A client comes to the crisis intervention clinic and tearfully tells the nurse, "It is so painful! I have thought about it, and I cannot see how I can go on without my partner." The nurse states, "You have resilience and will look back on this as a crisis you were able to manage." Analysis of this interaction reveals that the nurse A. has a good understanding of the effect of time on perception of a crisis. B. is offering a statement of positive outcome based on client coping ability. C. has not followed up on the client's verbal clues to suicidal thoughts. D. has stepped into the territory of traditional psychotherapy.

C) has not followed up on the client's verbal clues to suicidal thoughts. Nurses who are uncomfortable with the idea of suicide may fail to pick up on a client's clues. This client clearly was open to discussing her suicidal thoughts, or she would not have said, "I cannot see how I can go on."

When the client begins to sob uncontrollably and her speech becomes so incoherent that she is unable to give the nurse any information, the immediate interventions will focus on A. securing hospital admission. B. contacting a family member or close friend. C. lowering her anxiety level from severe to moderate. D. assisting the client to identify new effective coping strategies.

C) lowering her anxiety level from severe to moderate. Individuals with severe anxiety are not able to collaborate in problem solving. The nurse must assist the client to lower anxiety from severe to moderate or lower.

A client is treated in the emergency department for injuries sustained while vacationing hundreds of miles away from home. To best meet the client's emotional needs, the nurse should A. arrange to hospitalize the client. B. refer the client for traditional psychotherapy for posttraumatic stress disorder. C. provide temporary support by arranging shelter and contacting the client's friends. D. suggest that contacting a victim support group would be more appropriate than crisis intervention

C) provide temporary support by arranging shelter and contacting the client's friends. When a client has no support system, the nurse may assume that role for a short time.

What is the desirable outcome for the orientation stage of a nurse-patient relationship? The patient will demonstrate behaviors that indicate a. self-responsibility and autonomy. b. a greater sense of independence. c. rapport and trust with the nurse. d. resolved transference.

C) rapport and trust with the nurse. Development of rapport and trust is necessary before the relationship can progress to the working phase. Behaviors indicating a greater sense of independence, self-responsibility, and resolved transference occur in the working phase.

A person intentionally overdosed on antidepressants. Which nursing diagnosis has the highestpriority? a. Powerlessness b. Social isolation c. Risk for suicide d. Compromised family coping

C) risk for suicide This diagnosis is the only one with life-or-death ramifications and is therefore of higherpriority than the other options.

Which disorder is an example of a culture-bound syndrome? a) Epilepsy b) Schizophrenia c) Running amok d) Major depressive disorder

C) running amok Culture-bound syndromes occur in specific sociocultural contexts and are easily recognized by people in those cultures. A syndrome recognized in parts of Southeast Asia is running amok, in which a person (usually a male) runs around engaging in furious, almost indiscriminate violent behavior.

Which comment best indicates that a patient perceived the nurse was caring? "My nurse a. always asks me which type of juice I want to help me swallow my medication." b. explained my treatment plan to me and asked for my ideas about how to make it better." c. spends time listening to me talk about my problems. That helps me feel like I am not alone." d. told me that if I take all the medicines the doctor prescribes, then I will get discharged sooner."

C) spends time listening to me talk about my problems. That helps me feel like I am not alone." Caring evidences empathetic understanding as well as competency. It helps change pain and suffering into a shared experience, creating a human connection that alleviates feelings of isolation. The distracters give examples of statements that demonstrate advocacy or giving advice.

A nurse uses the SAD PERSONS scale to interview a patient. This tool provides data relevant to a. current stress level. b. mood disturbance. c. suicide potential. d. level of anxiety.

C) suicide potential The SAD PERSONS tool evaluates 10 major risk factors in suicide potential: sex, age, depression, previous attempt, ethanol use, rational thinking loss, social supports lacking, organized plan, no spouse, and sickness. The tool does not have categories to provide information on the other options listed.

A woman comes to the crisis intervention clinic expressing concern that her 16-year-old is using illegal drugs in their home.The nurse will A. encourage the mother to call the police when her son brings drugs home. B. inform her of the obligation to report this information to the police. C. work with the client to set goals that are mutually acceptable. D. refer the client to the police for consultation.

C) work with the client to set goals that are mutually acceptable. Goal setting is a collaborative task. Goals must be acceptable to the client and seen by the nurse as safe and appropriate

A client is displaying symptomatology reflective of a panic attack. In order to help the client regain control, the nurse responds: A. "You need to calm yourself." B. "What is it that you would like me to do to help you?" C. "Can you tell me what you were feeling just before your attack?" D. "I will get you some medication to help calm you."

C. "Can you tell me what you were feeling just before your attack?" A response that helps the client identify the precipitant stressor is most therapeutic.

A peer asks you to help him differentiate between culture and ethnicity for clarification. Which statement by the peer would acknowledge that you had appropriately helped him clarify the difference between the two terms? A. "So, ethnicity refers to having the same life goals whereas culture refers to race." B. "So, ethnicity refers to norms within a culture, and culture refers to shared likes and dislikes." C. "So, ethnicity refers to shared history and heritage, whereas culture refers to sharing the same beliefs and values." D. "So, ethnicity refers to race, and culture refers to having the same worldview."

C. "So, ethnicity refers to shared history and heritage, whereas culture refers to sharing the same beliefs and values." Ethnicity is sharing a common history and heritage. Culture comprises the shared beliefs, values, and practices that guide a group's members in patterned ways of thinking and acting. The other options are all incorrect definitions of ethnicity and culture.

The psychiatric mental health nurse working with depressed clients of the Eastern culture must realize that a useful outcome criterion might be if client reports A. increased somatic expressions of distress. B. disruption of energy balance. C. appeasement of the spirits. D. increased anxiety

C. Appeasement of the spirits Appeasement of spirits might be a viable outcome criterion if the client believes the illness was caused by angry spirits. In each of the other options useful outcomes would be decreased somatic symptoms, reinstatement of energy balance, and decreased anxiety.

Which source of healing might be most satisfactory to a client who believes his illness is caused by spiritual forces? A. Acupuncture B. Dietary change C. Cleansings D. Herbal medicine

C. Cleansings Rituals, cleansings, prayer, and even witchcraft may be the treatment expectation of a client who believes his illness is caused by spiritual forces.

Which of the following scenarios describe a HIPAA violation? A. Janie, the ED nurse, gives report to Amanda, a nurse on the intensive care unit, regarding Joel, who is being admitted. B. Mark, a nurse on the medical-surgical floor, calls his patient's primary care provider to obtain a list of current medications. C. Lyla, a nurse on the cardiac unit, gives report to Chloe, the nurse on the step-down unit, regarding the patient Lyla, who will be transferring, while they are walking in the hospital hallway. D. Tony, a nurse on the psychiatric unit, gives discharge information to the counseling office where his patient will be going to outpatient treatment after discharge.

C. Lyla, a nurse on the cardiac unit, gives report to Chloe, the nurse on the step-down unit, regarding the patient Lyla, who will be transferring, while they are walking in the hospital hallway. Rationale: Discussing a patient's information in public places where it may be overheard is a violation of a patient's confidentiality. The other options describe appropriate interactions for patient continuity of care and support of the treatment plan by the health care team.

The intervention that will be most effective in preventing a nurse from making decisions that will lead to legal difficulties is: A. Asking a peer to review nursing intervention related decisions. B. Balancing the rights of the client and the rights of society. C. Maintaining currency in state laws affecting nursing practice. D. Seeking value clarification about fundamental ethical principles.

C. Maintaining currency in state laws affecting nursing practice. Rationale: Each nurse's practice is governed by the Nurse Practice Act of the state in which the nurse practices. The nurse should always be aware of its provisions.

The plan of care for a client who has elaborate washing rituals specifies that response prevention is to be used. Which scenario is an example of response prevention? A. Having the client repeatedly touch "dirty" objects B. Not allowing the client to seek reassurance from staff C. Not allowing the client to wash hands after touching a "dirty" object D. Telling the client that he or she must relax whenever tension mounts

C. Not allowing the client to wash hands after touching a "dirty" object Response prevention is a technique by which the client is prevented from engaging in the compulsive ritual. A form of behavior therapy, response prevention is never undertaken without physician approval.

After the death of a client, what rule of confidentiality should be followed by nurses who provided care for the individual? A. Confidentiality is now reserved to the immediate family. B. Only HIV status continues to be protected and privileged. C. Nothing may be disclosed that would have been kept confidential before death. D. The nurse must confer with the next of kin before divulging confidential, sensitive information.

C. Nothing may be disclosed that would have been kept confidential before death. Rationale: Confidentiality extends to death and beyond. Nurses should never disclose information after the death of a client that they would have kept confidential while the client was alive.

Which of the following best explains the concept of cultural competence? A. Nurses have enough knowledge about different cultures to be assured they are delivering culturally sensitive care. B. Nurses are able to educate their patients from other cultures appropriately about the cultural norms of the United States. C. Nurses adjust their own practices to meet their patients' cultural preferences, beliefs, and practices. D. Nurses must take continuing education classes on culture in the process of becoming culturally competent.

C. Nurses adjust their own practices to meet their patients' cultural preferences, beliefs, and practices. Cultural competence means that nurses adjust and conform to their patients' cultural needs, beliefs, practices, and preferences rather than their own. This option does not describe cultural competence. Although nurses are continually learning regarding culture, it is a career-long process. The goal is not to educate patients about our own culture but rather to adjust to their cultural preferences. Although nurses may take continuing education regarding culture, this does not describe the term cultural competence. The other options do not describe cultural competence.

Which right of the client has been violated if he is medicated without being asked for his permission? A. Right to dignity and respect B. Right to treatment C. Right to informed consent D. Right to refuse treatment

C. Right to informed consent Rationale: Before being given medication, the client should be fully informed about the reason for, the expected outcomes of, and any side effects of the medication. The client has the right to refuse medication. If, in a nonemergency situation, he is given medication after refusing it, his right to informed consent has been violated.

If a client with psychiatric illness is determined to be incompetent to make decisions affecting his care A. Staff members are required to use their best judgment when defining care. B. No treatment other than custodial care can be provided. C. The court appoints a guardian to make decisions on his behalf. D. The doctrine of least restrictive alternative is null and void.

C. The court appoints a guardian to make decisions on his behalf. Rationale: An incompetent client is unable to make legal decisions that would affect his care, such as consenting to surgery. A court-appointed guardian functions on behalf of the client.

Lana is out of surgery and on the medical-surgical unit for recovery. You visit her the day after her surgical procedure. While you are in the room, Lana becomes visibly anxious and short of breath, and she states, "I feel so anxious! Something is wrong!" Your best action is to: A. reassure Lana that she is experiencing normal anxiety and do deep breathing exercises with her. B. use the call light to inquire whether Lana has any prn anxiety medication. C. call for help and assess Lana's vital signs. D. tell Lana you will stay with her until the anxiety subsides.

C. call for help and assess Lana's vital signs. In anxiety caused by a medical condition, the individual's symptoms of anxiety are a direct physiological result of a medical condition, such as hyperthyroidism, pulmonary embolism, or cardiac dysrhythmias. In this case Lana is postoperative and could be experiencing a pulmonary embolism, as evidenced by the shortness of breath and anxiety. She needs immediate evaluation for any serious medical condition. The other options would all be appropriate after it has been determined that no serious medical condition is causing the anxiety.

A Gulf War veteran is entering treatment for post-traumatic stress disorder. An important facet of assessment is to: A. ascertain how long ago the trauma occurred. B. find out if the client uses acting-out behavior. C. determine use of chemical substances for anxiety relief. D. establish whether the client has chronic hypertension related to high anxiety.

C. determine use of chemical substances for anxiety relief. Substance abuse often coexists with post-traumatic stress disorder. It is often the client's way of self-medicating to gain relief of symptoms.

When members of a group are introduced to the culture's worldview, beliefs, values, and practices, it is called A. acculturation. B. ethnocentrism. C. enculturation. D. cultural encounters.

C. enculturation. Members of a group are introduced to the culture's worldview, beliefs, values, and practices in a process called enculturation. Ethnocentrism is the universal tendency of humans to think that their way of thinking and behaving is the only correct and natural way. Acculturation is learning the beliefs, values, and practices of a new cultural setting, which sometimes takes several generations. Cultural encounters occur when members of varying cultures meet and interact.

Panic attacks in Latin American individuals often involve: A. repetitive involuntary actions. B. blushing. C. fear of dying. D. offensive verbalizations.

C. fear of dying. Panic attacks in Latin Americans and Northern Europeans often involve sensations of choking, smothering, numbness or tingling, as well as fear of dying.

Exclusive use of Western psychological theories by nurses making client assessments will result in A. a high level of care for all clients. B. standardization of nomenclature for psychiatric disorders. C. inadequate assessment of clients of diverse cultures. D. greater ease in selecting appropriate treatment interventions.

C. inadequate assessment of clients of diverse cultures. Unless clients have faith in a particular healing modality, the treatment may not be effective. When nurses make assessments on the basis of Western theories, treatments consistent with those assessments follow. Clients of other cultures may find the treatment modalities unacceptable or not useful. Treatments consistent with the client's cultural beliefs as to what will provide a cure are better.

Clients of another culture are at greatest risk for misdiagnosis of a psychiatric problem because of A. biased assessment tools. B. insensitive practitioners. C. insensitive interviewing techniques. D. lack of the availability of cultural translators.

C. insensitive interviewing techniques. Inaccurate information or insufficient information may be obtained if the interviewer is not culturally sensitive. Only when assessment data are accurate can effective treatment be planned.

If a client's record mentions that the client habitually relies on rationalization, the nurse might expect the client to: A. make jokes to relieve tension. B. miss appointments. C. justify illogical ideas and feelings. D. behave in ways that are the opposite of his or her feelings.

C. justify illogical ideas and feelings. Rationalization involves justifying illogical or unreasonable ideas or feelings by developing logical explanations that satisfy the teller and the listener.

A young adult applying for a position is mildly tense but eager to begin the interview. This can be assessed as showing: A. denial. B. compensation. C. normal anxiety. D. selective inattention.

C. normal anxiety. Normal anxiety is a healthy life force needed to carry out the tasks of living and striving toward goals. It prompts constructive actions.

A person who recently gave up smoking and now talks constantly about how smoking fouls the air, causes cancer, and "burns" money that could be better spent to feed the poor is demonstrating: A. projection. B. rationalization. C. reaction formation. D. undoing.

C. reaction formation. Reaction formation keeps unacceptable feelings or behaviors out of awareness by developing the opposite behavior or emotion.

A client frantically reports to the nurse that "You have got to help me! Something terrible is happening. I can't think. My heart is pounding, and my head is throbbing." The nurse should assess the client's level of anxiety as: A. mild. B. moderate. C. severe. D. panic.

C. severe. Severe anxiety is characterized by feelings of falling apart and impending doom, impaired cognition, and severe somatic symptoms such as headache and pounding heart.

When prescribed lorazepam (Ativan) 1 mg po qid for 1 week for generalized anxiety disorder, the nurse should: A. question the physician's order because the dose is excessive. B. explain the long-term nature of benzodiazepine therapy. C. teach the client to limit caffeine intake. D. tell the client to expect mild insomnia.

C. teach the client to limit caffeine intake. Caffeine is an antagonist of antianxiety medication.

A citizen at a community health fair asks the nurse, "What is the most prevalent mental disorder in the United States?" Select the nurse's correct response. a) Schizophrenia b) Bipolar disorder c) Dissociative fugue d) Alzheimer's disease

D) Alzheimers Disease The 12-month prevalence for Alzheimer's disease is 10% for persons older than 65% and 50% for persons older than 85. The prevalence of schizophrenia is 1.1% per year. The prevalence of bipolar disorder is 2.6%. Dissociative fugue is a rare disorder.

A nurse is reviewing confidentiality requirements with a newly licensed nurse. Which of the following statements by the newly licensed nurse indicates an understanding of the information? A) "I am legally required to notify a client's employer about a substance use disorder" B) "If a client is involuntarily committed, I can discuss information with the client's next of kin" C) "I can discuss a client's treatment with others as long as they are employees of the facility" D) "I should keep information private even after a client dies"

D) "I should keep information private even after a client dies

Which remark by a patient indicates passage from orientation to the working phase of a nurse-patient relationship? a. "I don't have any problems." b. "It is so difficult for me to talk about problems." c. "I don't know how it will help to talk to you about my problems." d. "I want to find a way to deal with my anger without becoming violent."

D) "I want to find a way to deal with my anger without becoming violent." Thinking about a more constructive approach to dealing with anger indicates a readiness to make a behavioral change. Behavioral change is associated with the working phase of the relationship. Denial is often seen in the orientation phase. It is common early in the relationship, before rapport and trust are firmly established, for a patient to express difficulty in talking about problems. Stating skepticism about the effectiveness of the nurse-patient relationship is more typically a reaction during the orientation phase.

A nurse counsels a patient with recent suicidal ideation. Which is the nurse's most therapeutic comment? a. "Let's make a list of all your problems and think of solutions for each one." b. "I'm happy you're taking control of your problems and trying to find solutions." c. "When you have bad feelings, try to focus on positive experiences from your life." d. "Let's consider which problems are very important and which are less important."

D) "Let's consider which problems are very important and which are less important." The nurse helps the patient develop effective coping skills. Assist the patient to reduce the overwhelming effects of problems by prioritizing them. The incorrect options continue to present overwhelming approaches to problem solving.

Which comment best indicates a patient is self-actualized? a. "I have succeeded despite a world filled with evil." b. "I have a plan for my life. If I follow it, everything will be fine." c. "I'm successful because I work hard. No one has ever given me anything." d. "My favorite leisure is walking on the beach, hearing soft sounds of rolling waves."

D) "My favorite leisure is walking on the beach, hearing soft sounds of rolling waves." The self-actualized personality is associated with high productivity and enjoyment of life. Self-actualized persons experience pleasure in being alone and an ability to reflect on events.

Select the best response for the nurse to a question from another health professional regarding the difference between a diagnosis in DSM-V and a nursing diagnosis. a. "There is no functional difference between the two. Both identify human disorders." b. "The DSM-V diagnosis disregards culture, whereas the nursing diagnosis takes culture into account." c. "The DSM-V diagnosis describes causes of disorders whereas a nursing diagnosis does not explore etiology." d. "The DSM-V diagnosis guides medical treatment, whereas the nursing diagnosis offers a framework for identifying interventions for issues a patient is experiencing."

D) "The DSM-V diagnosis guides medical treatment, whereas the nursing diagnosis offers a framework for identifying interventions for issues a patient is experiencing." The medical diagnosis is concerned with the patient's disease state, causes, and cures, whereas the nursing diagnosis focuses on the patient's response to stress and possible caring interventions. Both tools consider culture. The DSM-V is multiaxial. Nursing diagnoses also consider potential problems.

Which nursing statement is an example of reflection? a. "I think this feeling will pass." b. "So you are saying that life has no meaning." c. "I'm not sure I understand what you mean." d. "You look sad."

D) "You look sad."

Which individual in the emergency department should be considered at highest risk for completing suicide? a. An adolescent Asian American girl with superior athletic and academic skills who has asthma b. A 38-year-old single, African American female church member with fibrocystic breast disease c. A 60-year-old married Hispanic man with 12 grandchildren who has type 2diabetes d. A 79-year-old single, white male diagnosed recently with terminal cancer of the prostate

D) A 79-year-old single, white male diagnosed recently with terminal cancer of the prostate High-risk factors include being an older adult, single, male, and having a co-occurring medical illness. Cancer is one of the somatic conditions associated with increased suicide risk.Protective factors for African American women and Hispanic individuals include strong religious and family ties. Asian Americans have a suicide rate that increases with age.

Which of the following terms is used to describe a patient's fear of the outdoors? A) Xenophobia B) Mysophobia C) Acrophobia D) Agoraphobia

D) Agoraphobia xenopbobia= fear of strangers Mysophobia= fear of dirt/germs Acrophobia= fear of heights

A nurse in an acute mental health facility is leading a nursing staff discussion about the legal aspects of involuntary admission. Which of the following information should the nurse include? A) A client involuntarily admitted must take prescribed medications B) An involuntary admission of a client lasts up to 2 weeks C) A client who is involuntarily admitted can leave the facility against medical advice D) An involuntary admission is justified if the client is a threat to themselves and others

D) An involuntary admission is justified if the client is a threat to themselves and others

Therapeutic communication is the foundation of a patient- centered interview. Which of the following techniques is not considered therapeutic? a. Restating b. Encouraging description of perception c. Summarizing d. Asking "why" questions

D) Asking "why" questions

Which belief will best support a nurse's efforts to provide patient advocacy during a multidisciplinary patient care planning session? a. All mental illnesses are culturally determined. b. Schizophrenia and bipolar disorder are cross-cultural disorders. c. Symptoms of mental disorders are unchanged from culture to culture. d. Assessment findings in mental illness reflect a person's cultural patterns.

D) Assessment findings in mental illness reflect a person's cultural patterns

A 26-month-old displays negative behavior, refuses toilet training, and often says, "No!" Which psychosocial crisis is evident? a. Trust versus mistrust b. Initiative versus guilt c. Industry versus inferiority d. Autonomy versus shame and doubt

D) Autonomy versus shame and doubt The crisis of autonomy versus shame and doubt relates to the developmental task of gaining control of self and environment, as exemplified by toilet training. This psychosocial crisis occurs during the period of early childhood. Trust versus mistrust is the crisis of the infant. Initiative versus guilt is the crisis of the preschool and early-school-aged child. Industry versus inferiority is the crisis of the 6- to 12-year-old child.

Which aspect of direct care is an experienced, inpatient psychiatric nurse most likely to provide for a patient? a. Hygiene assistance b. Diversional activities c. Assistance with job hunting d. Building assertiveness skills

D) Building assertiveness skills Assertiveness training relies on the counseling and psychoeducational skills of the nurse. Assistance with personal hygiene would usually be accomplished by a psychiatric technician or nursing assistant. Diversional activities are usually the province of recreational therapists. The patient would probably be assisted in job hunting by a social worker or vocational therapist.

A nurse is part of a multidisciplinary team working with groups of depressed patients. One group of patients receives supportive interventions and antidepressant medication. The other group receives only medication. The team measures outcomes for each group. Which type of study is evident? a. Incidence b. Prevalence c. Comorbidity d. Clinical epidemiology

D) Clinical Epidemiology Clinical epidemiology is a broad field that addresses studies of the natural history (or what happens if there is no treatment and the problem is left to run its course) of an illness, studies of diagnostic screening tests, and observational and experimental studies of interventions used to treat people with the illness or symptoms. Prevalence refers to numbers of new cases. Comorbidity refers to having more than one mental disorder at a time. Incidence refers to the number of new cases of mental disorders in a healthy population within a given period. See related audience response question.

A patient says, "People should be allowed to commit suicide without interference from others." A nurse replies, "You're wrong. Nothing is bad enough to justify death." What is the best analysis of this interchange? a. The patient is correct. b. The nurse is correct .c. Neither person is correct. d. Differing values are reflected in the two statements.

D) Differing values are reflected in the two statements. Values guide beliefs and actions. The individuals stating their positions place different values on life and autonomy. Nurses must be aware of their own values and be sensitive to the values of others.

A nurse is preparing to apply wrist restraints on a client who is threatening to harm others and has not responded to less invasive interventions. Which of the following actions should the nurse plan to take? A) Obtain a PRN prescription for the restraints for the client's provider B) Visually observe the client every 10 minutes until restraints are removed C) Ensure that three fingers can fit between the restraint and the client's wrist D) Document the client's behavior every 15 min while restraints are in place

D) Document the client's behavior every 15 min while the restraints are in place Restraints are NEVER PRN only 2 fingers

A nurse is administering an oral sedative to a client who is receiving care following an involuntary admission. The client states, "I am not taking any more medication." Which of the following actions should the nurse take? A) Administer the medication by another route B) Refer the client's refusal to the facility's ethics committee C) Inform the client that, due to her involuntary admission, they cannnot refuse a sedative D) Document the client's refusal of the medication in their medical record

D) Document the client's refusal of the medication in their medical record

Which therapies involve electrical brain stimulation for treatment of mental illness? (Select all that apply.) a. Aversion therapy b. Operant conditioning c. Systematic desensitization d. Electroconvulsive therapy (ECT) e. Transcranial magnetic stimulation (TMS)

D) Electroconvulsive therapy (ECT) E) Transcranial magnetic stimulation (TMS) ECT and TMS are therapies that use electrical stimulation of the brain as a form of treatment for mental illness. The incorrect responses are therapies that are interpersonal in nature.

A patient's relationships are intense and unstable. The patient initially idealizes the significant other and then devalues him or her, resulting in frequent feelings of emptiness. This patient will benefit from interventions to develop which aspect of mental health? a. Effectiveness in work b. Communication skills c. Productive activities d. Fulfilling relationships

D) Fulfilling relationships The information given centers on relationships with others that are described as intense and unstable. The relationships of mentally healthy individuals are stable, satisfying, and socially integrated. Data are not present to describe work effectiveness, communication skills, or activities.

A nurse listens to a group of recent retirees. One says, "I volunteer with Meals on Wheels, coach teen sports, and do church visitation." Another laughs and says, "I'm too busy taking care of myself to volunteer to help others." Which psychosocial developmental task do these statements contrast? a. Trust and mistrust b. Intimacy and isolation c. Industry and inferiority d. Generativity and self-absorption

D) Generativity and self-absorption Both retirees are in middle adulthood, when the developmental crisis to be resolved is generativity versus self-absorption. One exemplifies generativity; the other embodies self-absorption. This developmental crisis would show a contrast between relating to others in a trusting fashion and being suspicious and lacking trust. Failure to negotiate this developmental crisis would result in a sense of inferiority or difficulty learning and working as opposed to the ability to work competently. Behaviors that would be contrasted would be emotional isolation and the ability to love and commit oneself.

Which measure would be considered a form of primary prevention for suicide? a. Psychiatric hospitalization of a suicidal patient b. Referral of a formerly suicidal patient to a support group c. Suicide precautions for 24 hours for newly admitted patients d. Helping school children learn to manage stress and be resilient

D) Helping school children learn to manage stress and be resilient This measure promotes effective coping and reduces the likelihood that such children will become suicidal later in life. Admissions and suicide precautions are secondary prevention measures. Support group referral is a tertiary prevention measure.

A person says, "I was the only survivor in a small plane crash. Three business associates died. I got depressed and saw a counselor twice a week for 4 weeks. We talked about my feelings related to being a survivor, and I'm better now." Which type of therapy was used? a. Systematic desensitization b. Psychoanalysis c. Behavior modification d. Interpersonal psychotherapy

D) Interpersonal psychotherapy Interpersonal psychotherapy returned the patient to his former level of functioning by helping him come to terms with the loss of friends and guilt over being a survivor. Systematic desensitization is a type of therapy aimed at extinguishing a specific behavior, such as the fear of flying. Psychoanalysis would call for a long period of exploration of unconscious material. Behavior modification would focus on changing a behavior rather than helping the patient understand what is going on in his life.

Four individuals have given information about their suicide plans. Which plan evidences the highest suicide risk? a. Turning on the oven and letting gas escape into the apartment during the night b. Cutting the wrists in the bathroom while the spouse reads in the next room c. Overdosing on aspirin with codeine while the spouse is out with friends d. Jumping from a railroad bridge located in a deserted area late at night

D) Jumping from a railroad bridge located in a deserted area late at night This is a highly lethal method with little opportunity for rescue. The other options are lower lethality methods with higher rescue potential. See relationship to audience response question.

Crises that occur as an individual moves from one developmental level to another are called A. reactive crises. B. recurring crises. C. situational crises. D. maturational crises.

D) Maturational Crises Maturational crises are normal states in growth and development in which specific new maturational tasks must be learned when old coping mechanisms are no longer effective.

The DSM-V classifies: a) deviant behaviors. b) present disability or distress. c) people with mental disorders. d) mental disorders people have.

D) Mental disorders people have The DSM-V classifies disorders people have rather than people themselves. The terminology of the tool reflects this distinction by referring to individuals with a disorder rather than as a "schizophrenic" or "alcoholic," for example. Deviant behavior is not generally considered a mental disorder. Present disability or distress is only one aspect of the diagnosis.

A patient expresses a desire to be cared for by others and often behaves in a helpless fashion. Which stage of psychosexual development is most relevant to the patient's needs? a. Latency b. Phallic c. Anal d. Oral

D) Oral Fixation at the oral stage sometimes produces dependent infantile behaviors in adults. Latency fixations often result in difficulty identifying with others and developing social skills, resulting in a sense of inadequacy and inferiority. Phallic fixations result in having difficulty with authority figures and poor sexual identity. Anal fixation sometimes results in retentiveness, rigidity, messiness, destructiveness, and cruelty. This item relates to an audience response question.

Which nursing intervention below is part of the scope of an advanced practice psychiatric/mental health nurse rather than a basic level registered nurse? a. Coordination of care b. Health teaching c. Milieu therapy d. Psychotherapy

D) Psychotherapy Psychotherapy is part of the scope of practice of an advanced practice nurse. The distracters are within a basic level registered nurse's scope of practice.

A nurse supports a parent for praising a child who behaves in helpful ways to others. When this child behaves with politeness and helpfulness in adulthood, which feeling will most likely result? a. Guilt b. Anxiety c. Humility d. Self-esteem

D) Self-esteem The individual will be living up to the ego ideal, which will result in positive feelings about self. The other options are incorrect because each represents a negative feeling.

A nurse wants to find information on current evidence-based research, programs, and practices regarding mental illness and addictions. Which resource should the nurse consult? a. American Psychiatric Association b. American Psychological Association (APA) c. Clinician's Quick Guide to Interpersonal Psychotherapy d. Substance Abuse and Mental Health Services Administration (SAMHSA)

D) Substance Abuse and Mental Health Services Administration (SAMHSA) The SAMHSA maintains a National Registry of Evidence-based Practices and Programs. New therapies are entered into the database on a regular basis. The incorrect responses are resources but do not focus on evidence-based information.

Which individual is demonstrating the highest level of resilience? One who a) is able to repress stressors. b) becomes depressed after the death of a spouse. c) lives in a shelter for 2 years after the home is destroyed by fire. d) takes a temporary job to maintain financial stability after loss of a permanent job.

D) Takes a temporary job to maintain financial stability after loss of a permanent job Resilience is closely associated with the process of adapting and helps people facing tragedies, loss, trauma, and severe stress. It is the ability and capacity for people to secure the resources they need to support their well-being. Repression and depression are unhealthy. Living in a shelter for 2 years shows a failure to move forward after a tragedy. See related audience response question.

A health care provider prescribed long acting antipsychotic medication injections every 3 weeks at the clinic for a patient with a history of medication nonadherence. For this plan to be successful, which factor will be of critical importance? a. The attitude of significant others toward the patient b. Nutrition services in the patient's neighborhood c. The level of trust between the patient and nurse d. The availability of transportation to the clinic

D) The availability of transportation to the clinic The ability of the patient to get to the clinic is of paramount importance to the success of the plan. The long acting antipsychotic medication injections relieve the patient of the necessity to take medication daily, but if he or she does not receive the injection at 3-week intervals, non-adherence will again be the issue. Attitude toward the patient, trusting relationships, and nutrition are important but not fundamental to this particular problem.

Which client-focused scenario best demonstrates an example of eustress? A) the client receives notification that their employer is experiencing financial problems and some workers will be terminated B) The client receives a bank notice that there were insufficient funds in their account for a recent payment C) the client loses a beloved family pet D) The client prepares to take a vacation to a tropical island with a group of close friends

D) The client prepares to take a vacation to a tropical island with a group of close friends

In the event of an adventitious crisis, which age group would be least in need of crisis intervention? A. Children B. Such crises seldom require intervention C. The elderly D. A distinction cannot be made

D) a distinction cannot be made The need for psychological first aid (crisis intervention) and debriefing after any crisis situation cannot be overstressed for all age groups (children, adolescents, adults, and the elderly).

A nurse caring for a withdrawn, suspicious patient recognizes development of feelings of anger toward the patient. The nurse should a. suppress the angry feelings. b. express the anger openly and directly with the patient. c. tell the nurse manager to assign the patient to another nurse. d. discuss the anger with a clinician during a supervisory session.

D) discuss the anger with a clinician during a supervisory session. The nurse is accountable for the relationship. Objectivity is threatened by strong positive or negative feelings toward a patient. Supervision is necessary to work through countertransference feelings.

A person who attempted suicide by overdose was treated in the emergency department andthen hospitalized. The initial outcome is that the patient will a. verbalize a will to live by the end of the second hospital day. b. describe two new coping mechanisms by the end of the third hospital day. c. accurately delineate personal strengths by the end of first week of hospitalization. d. exercise suicide self-restraint by refraining from attempts to harm self for 24 hours.

D) exercise suicide self-restraint by refraining from attempts to harm self for 24 hours. Suicide self-restraint relates most directly to the priority problem of risk for self-directedviolence. The other outcomes are related to hope, coping, and self-esteem.

Tori knows that Carter needs assistance with many aspects of getting through the crisis. Tori's highest priority in Carter's care is: A. reduction of Carter's anxiety. B. development of new coping skills. C. prevention of boundary blurring. D. keeping Carter safe.

D) keeping Carter safe. The nurse's initial task is to promote safety by assessing the patient's potential for suicide or homicide. The other options are all important components of the care plan, but safety of the patient takes the highest priority.

The nurse assigned to ACT should explain the program's treatment goal as a. assisting patients to maintain abstinence from alcohol and other substances of abuse. b. providing structure and a therapeutic milieu for mentally ill patients whose symptoms require stabilization. c. maintaining medications and stable psychiatric status for incarcerated inmates who have a history of mental illness. d. providing services for mentally ill individuals who require intensive treatment to continue to live in the community.

D) providing services for mentally ill individuals who require intensive treatment to continue to live in the community. An ACT program provides intensive community services to persons with serious, persistent mental illness who live in the community but require aggressive services to prevent repeated hospitalizations.

Which patient would be most appropriate to refer for assertive community treatment (ACT)? A patient diagnosed with a. a phobic fear of crowded places. b. a single episode of major depressive disorder. c. a catastrophic reaction to a tornado in the community. d. schizophrenia and four hospitalizations in the past year.

D) schizophrenia and four hospitalizations in the past year. ACT provides intensive case management for persons with serious persistent mental illness who live in the community. Repeated hospitalization is a frequent reason for this intervention. The distracters identify mental health problems of a more episodic nature.

The nurse should refer which of the following patients to a partial hospitalization program? A patient who a. has a therapeutic lithium level and reports regularly for blood tests and clinic follow-up. b. needs psychoeducation for relaxation therapy related to agoraphobia and panic episodes. c. spent yesterday in a supervised crisis care center and continues to have active suicidal ideation. d. states, "I'm not sure I can avoid using alcohol when my spouse goes to work every morning."

D) states, "I'm not sure I can avoid using alcohol when my spouse goes to work every morning." This patient could profit from the structure and supervision provided by spending the day at the partial hospitalization program. During the evening, at night, and on weekends, the spouse could assume responsibility for supervision. A suicidal patient needs inpatient hospitalization. The other patients can be served in the community or with individual visits.

Clinical pathways are used in managed care settings to a. stabilize aggressive patients. b. identify obstacles to effective care. c. relieve nurses of planning responsibilities. d. streamline the care process and reduce costs.

D) streamline the care process and reduce costs. Clinical pathways provide guidelines for assessments, interventions, treatments, and outcomes as well as a designated timeline for accomplishment. Deviations from the timeline must be reported and investigated. Clinical pathways streamline the care process and save money. Care pathways do not identify obstacles or stabilize aggressive patients. Staff are responsible for the necessary interventions. Care pathways do not relieve nurses of the responsibility of planning; pathways may, however, make the task easier.

A nurse is talking with a patient, and 5 minutes remain in the session. The patient has been silent most of the session. Another patient comes to the door of the room, interrupts, and says to the nurse, "I really need to talk to you." The nurse should a. invite the interrupting patient to join in the session with the current patient. b. say to the interrupting patient, "I am not available to talk with you at the present time." c. end the unproductive session with the current patient and spend time with the interrupting patient. d. tell the interrupting patient, "This session is 5 more minutes; then I will talk with you."

D) tell the interrupting patient, "This session is 5 more minutes; then I will talk with you." When a specific duration for sessions has been set, the nurse must adhere to the schedule. Leaving the first patient would be equivalent to abandonment and would destroy any trust the patient had in the nurse. Adhering to the contract demonstrates that the nurse can be trusted and that the patient and the sessions are important. The incorrect responses preserve the nurse-patient relationship with the silent patient but may seem abrupt to the interrupting patient, abandon the silent patient, or fail to observe the contract with the silent patient.

The patients below were evaluated in the emergency department. The psychiatric unit has one bed available. Which patient should be admitted? The patient a. feeling anxiety and a sad mood after separation from a spouse of 10 years. b. who self-inflicted a superficial cut on the forearm after a family argument. c. experiencing dry mouth and tremor related to taking antipsychotic medication. d. who is a new parent and hears voices saying, "Smother your baby."

D) who is a new parent and hears voices saying, "Smother your baby." Admission to the hospital would be justified by the risk of patient danger to self or others. The other patients have issues that can be handled with less restrictive alternatives than hospitalization.

The question that would give data of least value to the assessment of family dynamics is A. "What changes have occurred recently at work?" B. "Are your wife and children conforming to your expectations?" C. "Are you experiencing stress associated with conforming to family expectations?" D. "Do you expect others to shun or avoid you because you are seeing a therapist?"

D. "Do you expect others to shun or avoid you because you are seeing a therapist?" The question about others' reaction to seeking help from a psychotherapist will not provide data about family dynamics.

You are working on the psychiatric unit and assisting with the care for Mr. Tran, a refugee from Darfur, who came to the United States 1 year ago. Although Mr. Tran understands and speaks some very limited English, he is much more comfortable conversing in his native language. Mike, the nurse working directly with Mr. Tran, says to you, "I am so frustrated trying to communicate with Mr. Tran! He insists on speaking his language instead of English. I think if people want to live here, they ought to have to speak our language and act like we do!" Which of the following responses you could make promotes culturally competent care? (select all that apply): A. "You are right that Mr. Tran needs to speak English, but all patients do have a right to an interpreter, so you need to comply." B. "I agree that it is frustrating trying to communicate with Mr. Tran. Maybe we could see if his family members can help convince him to try speaking English." C. "Mr. Tran will have to learn to speak English eventually to live and work successfully in this country. Just try to be patient and encourage him to try speaking English." D. "What you are saying is actually considered cultural imposition, which is imposing our own culture onto someone from a different culture." E. "Mr. Tran's ability to speak and understand English is very limited. He needs to have an interpreter to make sure he can make his needs and feelings known.

D. "What you are saying is actually considered cultural imposition, which is imposing our own culture onto someone from a different culture." E. "Mr. Tran's ability to speak and understand English is very limited. He needs to have an interpreter to make sure he can make his needs and feelings known. Cultural imposition is imposing our own cultural norms onto those from another cultural group. By obtaining an interpreter for Mr. Tran, the nurse is promoting culturally competent care, ensuring the patient can communicate his feelings and needs thoroughly to the staff. Patients do have a right to an interpreter, but stating that Mike is right is not promoting culturally competent care and is instead confirming his opinion. Asking family members to convince the patient to speak English is not promoting culturally competent care and also undermines the trust between nurse and patient. Instead of encouraging the patient to speak English an interpreter should be obtained for the patient.

Which nursing intervention would be helpful when caring for a client diagnosed with an anxiety disorder? A. Express mild amusement over symptoms. B. Arrange for client to spend time away from others. C. Advise client to minimize exercise to conserve endorphins. D. Reinforce use of positive self-talk to change negative assumptions.

D. Reinforce use of positive self-talk to change negative assumptions. This technique is a variant of cognitive restructuring. "I can't do that" is changed to "I can do it if I try."

When assessing and planning treatment for a client who has recently arrived in the United States from China, the nurse should be alert to the possibility that the client's explanatory model for his illness reflects A. supernatural causes. B. negative forces. C. inheritance. D. imbalance.

D. Imbalance Many Eastern cultures explain illness as a function of imbalance.

Which statement is true regarding mail sent to an involuntarily admitted client residing on a psychiatric inpatient unit? A. The client can receive mail from only family and legal sources. B. Mail must first be opened and inspected by staff. C. Receipt of mail is considered a privilege accorded the client for compliance. D. Mail is a form of social interaction and so receiving mail is a client's civil right.

D. Mail is a form of social interaction and so receiving mail is a client's civil right. Rationale: The client's civil rights are intact, despite hospitalization. The right to communicate with those outside the hospital is ensured.

Which medication is FDA approved for treatment of anxiety in children? A. Lorazepam (benzodiazepine) B. Fluoxetine (selective serotonin reuptake inhibitor) C. Clomipramine (tricyclic antidepressant) D. None of the above

D. None of the above There are no medications with FDA approval for children with anxiety disorders; however, medications approved for other age groups are often prescribed. None of the other options are FDA approved to treat anxiety in children (see the previous sentence).

Which behavior would be characteristic of an individual who is displacing anger? A. Lying B. Stealing C. Slapping D. Procrastinating

D. Procrastinating A passive-aggressive person deals with emotional conflict by indirectly and unassertively expressing aggression toward others. Procrastination is an expression of resistance.

A client reports to the nurse that once he's released he will make sure his wife will never again be able to have him committed to a psychiatric hospital. What action should the nurse take? A. None, because no explicit threat has been made. B. Ask the client if he is threathening his wife. C. Call the client's wife and report the threat. D. Report the incident to the client's therapist and document.

D. Report the incident to the client's therapist and document. Rationale: The Tarasoff ruling makes it necessary for nurses to report client statements that imply the client may harm another person or persons. The nurse reports to the treatment team, and the mandated reporter (usually the professional leader of the team) is responsible for notifying the person against whom the threat was made.

Which healing practice is least used in the Western health system of healing practices? A. Antibiotic medication B. Surgery C. Targeted cellular destruction D. Restoring lost balance or harmony

D. Restoring lost balance or harmony The best treatment perspectives of various cultures include regaining lost balance and harmony. This perspective is not used in Western culture.

What assumption can be made about the client who has been admitted on an involuntary basis? A. The client can be discharged from the unit on demand. B. For the first 48 hours, the client can be given medication over objection. C. The client has agreed to fully participate in treatment and care planning. D. The client is a danger to self or others or unable to meet basic needs.

D. The client is a danger to self or others or unable to meet basic needs. Rationale: Involuntary admission implies that the client did not consent to the admission. The usual reasons for admitting a client over his or her objection is if the client presents a clear danger to self or others or is unable to meet even basic needs independently.

The nurse caring for a client experiencing a panic attack anticipates that the psychiatrist would order a stat dose of: A. standard antipsychotic medication. B. tricyclic antidepressant medication. C. anticholinergic medication. D. a short-acting benzodiazepine medication.

D. a short-acting benzodiazepine medication. A short-acting benzodiazepine is the only type of medication listed that would lessen the client's symptoms of anxiety within a few minutes. Anticholinergics do not lower anxiety; tricyclic antidepressants have very little antianxiety effect and have a slow onset of action; and a standard antipsychotic medication will lower anxiety but has a slower onset of action and the potential for more side effects.

People who have an indigenous worldview A. see themselves as spiritual and believe that they are linked with all other living things. B. focus on the articulation of individual needs and ideas. C. view the self as an extension of cosmic energy that is repeatedly reborn. D. are concerned with being part of a harmonious community.

D. are concerned with being part of a harmonious community. Clients with an indigenous worldview are interested in connectedness and being in harmony with others. They have little interest in personal goals and autonomy.

Working to help the client view an occurrence in a more positive light is called: A. flooding. B. desensitization. C. response prevention. D. cognitive restructuring.

D. cognitive restructuring. The purpose of cognitive restructuring is to change the individual's negative view of an event or a situation to a view that remains consistent with the facts but that is more positive.

The primary purpose of performing a physical examination before beginning treatment for any anxiety disorder is to: A. protect the nurse legally. B. establish the nursing diagnoses of priority. C. obtain information about the client's psychosocial background. D. determine whether the anxiety is primary or secondary in origin.

D. determine whether the anxiety is primary or secondary in origin. The symptoms of anxiety can be caused by a number of physical disorders or are said to be caused by an underlying physical disorder. The treatment for secondary anxiety is treatment of the underlying cause.

A symptom commonly associated with panic attacks is: A. obsessions. B. apathy. C. fever. D. fear of impending doom.

D. fear of impending doom. The feelings of terror present during a panic attack are so severe that normal function is suspended, the perceptual field is severely limited, and misinterpretation of reality may occur.

Deviation from cultural expectations is considered by members of the cultural group as a demonstration of A. hostility. B. lack of self-will. C. variation from tradition. D. illness.

D. illness. Deviation from cultural expectations is considered by others in the culture to be a problem and is frequently defined by the cultural group as "illness."

You are providing teaching to Lana, a preoperative patient just before surgery. She is becoming more and more anxious as you talk. She begins to complain of dizziness and heart pounding, and she is trembling. She seems confused. Your best response is to: A. reinforce the preoperative teaching by restating it slowly. B. have Lana read the teaching materials instead of verbal instruction. C. have a family member read the preoperative materials to Lana. D. not attempt any teaching at this time.

D. not attempt any teaching at this time. Patients experiencing severe anxiety, as the symptoms suggest, are unable to learn or solve problems. The other options would not be effective because you are still attempting to teach someone who has a severe level of anxiety.

Delusionary thinking is a characteristic of: A. chronic anxiety. B. acute anxiety. C. severe anxiety. D. panic level anxiety.

D. panic level anxiety. Panic level anxiety is the most extreme level and results in markedly disturbed thinking.

A client is running from chair to chair in the solarium. He is wide-eyed and keeps repeating, "They are coming! They are coming!" He neither follows staff direction nor responds to verbal efforts to calm him. The level of anxiety can be assessed as: A. mild. B. moderate. C. severe. D. panic.

D. panic. Panic-level anxiety results in markedly disorganized, disturbed behavior, including confusion, shouting, and hallucinating. Individuals may be unable to follow directions and may need external limits to ensure safety.

Venlafaxine (Effexor) exerts its antidepressant effect by selectively blocking the reuptake of: A. GABA B. Dopamine C. Serotonin D. Norepinephrine E. C and D

E. C and D


संबंधित स्टडी सेट्स

Hardware + Network Troubleshooting

View Set

Chapter 17-Somatic Symptoms Disorder

View Set

Altering the Environment Practice

View Set

ATI Care of Children Practice Assessment B 2017

View Set

Chapter 29 Chest injuries EMT School

View Set